Wednesday, May 06, 2015

Do you believe in magic

  • I don't believe that reason could arise from nonreason, therefore I think that reason is at the foundation of the universe. According to the naturalistic view, the normative arises from the nonnormative, the logical arises from the nonlogical, the universe exists without an explanation for its existence even though it looks contingent as all heck, the universe was finely tuned for intelligent life, purposes arise where none existed before, consciousness comes from a lack of consciousness. The very foundations of science don't even seem possible in the irrational universe that atheists believe in. Even the very fact that our thoughts are about something else is something that can't be captured by basic physics. It has always seemed to me that the atheists, not the theists, are the ones who believe in magic.


300 comments:

«Oldest   ‹Older   201 – 300 of 300
Unknown said...

@grodrigues: "maybe you are in the wrong place?"

Maybe. Where I'm from, when you make statements about a billion people's views being irrational, magical, and inconsistent with science, just because they happen to disagree with yours, you need to back those statements up.

grodrigues said...

@Wizard Suth:

"Where I'm from, when you make statements about a billion people's views being irrational, magical, and inconsistent with science, just because they happen to disagree with yours, you need to back those statements up."

And we have a buffoon in our midst. How fortunate we are.

planks length said...

Is your enjoyment of a symphony impaired by the knowledge that it will end?

Actually (and in complete honesty - not just to argue with you), yes it is. I love music, but I always feel that it has (at least in this life) two great failings:

1) I can only hear one note at a time.
2) It ends.

One of the joys of the next world will be to be able to appreciate heavenly music in its totality (all the notes at once), and that it will be eternal.

Ilíon said...

"And we have a buffoon in our midst. How fortunate we are."

Not to worry -- any second now, Jeffery Jay Lowder is going to Take A Stand For Reason And Civility And Mutual Respect (tm)

B. Prokop said...

"I notice you didn't refute his point."

And I notice that whenever a gnu wants to derail a conversation, he always mentions something that someone didn't say.

Hey Wizard, did it ever occur to you that it is impossible for each and every blog posting to contain the sum total of human knowledge. Good grief, deal with what Daniel said - not with what he didn't say!

Jezu ufam tobie!

Daniel Joachim said...

"I notice you didn't refute his point. You just attempted to dodge the question by citing an alleged authority.

A general statement such as "reason cannot arise from unreason" can be disproven with a single counter-example. Why isn't David Brightly's counter-example sufficient? It appears to be an example of reason arising from non-reason."


Yes, with the probable hypothesis that I was scared, and trying to do some serious waving of hands.

But ah, I did, implicitly, in the statement of Chastek. As would be obvious to anyone with a basic knowledge of act and potency.

More explicit. Babies are not an example of reason arising from non-existent reason. It's an example of a rational animal, actualizing his potential use of reason, that was really there all along. As you may know: Out of nothing, nothing comes.

However, you may know that Aristotelians have a very specific thing in mind when talking about "reason". If we're not clear on that, then this will (as I see in your dialogues above), be another rehearsal in talking past each other.

It's something unique to the genus of rational animals, including human babies. As well as immaterial.
http://www3.nd.edu/~afreddos/courses/43151/ross-immateriality.pdf

Unknown said...

@planks length: "One of the joys of the next world will be to be able to appreciate heavenly music in its totality (all the notes at once), and that it will be eternal."

That's a lovely fantasy, but I wouldn't count on it coming true. Heaven is the carrot religions use to keep people in line, just as Hell is the stick. Clergy will readily make grandiose promises that they have no ability to keep as long as they can find people gullible enough to listen.

Unknown said...

@Daniel Joachim: "Out of nothing, nothing comes."

Actually that is debatable. It depends largely on what you mean by nothing. If you mean empty space, particles pop into and out of existence spontaneously all the time. Oops. More physics.

It's also very difficult to prove either way, since we don't have an example of nothing to study. At best we can say that something can come from something, and that we don't know of any examples of something that did not come from something.

I would argue that the potential to have an ability (such as the ability to reason) is not the same as actually having that ability.

Ilíon said...

Daniel Joachim: "More explicit. Babies are not an example of reason arising from non-existent reason. It's an example of a rational animal, actualizing his potential use of reason, that was really there all along."

As witness: Helen Keller

Daniel Joachim said...

"Actually that is debatable. It depends largely on what you mean by nothing. If you mean empty space, particles pop into and out of existence spontaneously all the time. Oops. More physics."

Oh, that's beyond cute. Have you just read yourself some Krauss, and thought you could find some wriggle-room in an irrelevant (in this context) equivocal term of "nothing"? Grasping at straws?

What's next? Will you tell me that its debatable whether bats can fly, because they usually require a New York Yankee to swing them?

Oops. More good-ol' reality.

Of course, there's miles apart between actuality and potentiality, but that's irrelevant. We were talking about non-existence to existence.

Again, understanding what is actually meant by terms like "reason" or "potentiality", requires some basic schooling on what they mean in e.g. Aristotelianism, and why the distinctions are important. Otherwise, there's no point in having a dialogue, addressing only your intuitions.

Daniel Joachim said...

As witness: Helen Keller

Do you have anything in particular in mind? :)

planks length said...

Actually that is debatable. It depends largely on what you mean by nothing. If you mean empty space, particles pop into and out of existence spontaneously all the time. Oops. More physics.

Wizard, you really, desperately need to watch this talk by William Lane Craig. If you have even the least shred of intellectual honesty, then after viewing it you will never say anything as stupid as the above quote ever, ever again.

However, given your comments to date, I'm not holding my breath.

David Brightly said...

So 'I don't believe that reason could arise from nonreason' does not imply 'I have never seen an instance of reason arising from nonreason', because one clearly has. Rather it means something like, 'I have an argument that concludes with 'Reason cannot arise from nonreason', and I find this argument compelling'. Others will have examined the argument and not found it so compelling. Both sides will come up with further arguments to justify their positions. These arguments will all contain abstract terms whose meanings people will find hard to explain to one another, let alone come to agreement on. This is reason to doubt our faith in reason, or at least our faith in the use of reason in pursuit of certain sorts of philosophical inquiry.

Unknown said...

@Daniel Joachim: Thanks for the link.

I agree with some of the points James Ross makes:

1. A physical machine cannot perfectly simulate reasoning in all possible cases.

Granted. All physical machines have limits.

2. It may be impossible to distinguish something that is actually capable of reasoning from something that merely simulates it.

Okay. If you can't distinguish one from the other, why label them differently? Let's just call both of them reasoning, until we detect a difference. For the moment I'll go along for the ride.

The flaw in his argument comes when he supposes that since a physical machine cannot simulate reasoning perfectly in all cases, it cannot simulate reasoning accurately even in limited cases. He attempts to justify this by appealing to reason being a Platonic ideal which the simulation emulates, so that in order to show that its simulated reasoning is accurate (or inaccurate) you must (ultimately) appeal to something immaterial.

I would argue that "reason" is a label we apply to a set of processes that brains engage in, and which James Ross calls a simulation of reasoning. The Platonic ideals are also simulations that people use to model abstract concepts. We make models to help us interpret what we observe, and those models include the laws of logic, mathematics, and physics. The universe doesn't obey these laws because they were imposed by a cosmic law-giver. The laws were invented as models of how the universe works. They are descriptive, not prescriptive.

This was at least interesting. Do you have any better arguments?

Unknown said...

@planks length: "Wizard, you really, desperately need to watch this talk by William Lane Craig"

Ugh. I've seen enough of William Lane Craig's ill-founded assumptions and logical fallacies. The Kalam cosmological argument he presents is full of flaws. The "divine command theory" he espouses is reprehensible, because you can use it (as he does) to justify any atrocity by claiming that a god ordered it. He isn't even convinced by his own arguments, and ultimately appeals to his feelings (which he calls "the inner witness of the Holy Spirit") as his basis for belief.

But I'll watch the video anyway, just to humour you.

"after viewing it you will never say anything as stupid as the above quote ever, ever again."

I doubt that. My statement was based on one of the best-tested and most accurate theories in science, and one on which the technology I used to present it depends.

Unknown said...

@B. Prokop: Why do you refer to people as "gnus"? I will readily admit that I am not UNIX, but that doesn't imply that I am GNU.

Daniel Joachim said...

2. It may be impossible to distinguish something that is actually capable of reasoning from something that merely simulates it.

Okay. If you can't distinguish one from the other, why label them differently? Let's just call both of them reasoning, until we detect a difference. For the moment I'll go along for the ride.

The flaw in his argument comes when he supposes that since a physical machine cannot simulate reasoning perfectly in all cases, it cannot simulate reasoning accurately even in limited cases. He attempts to justify this by appealing to reason being a Platonic ideal which the simulation emulates, so that in order to show that its simulated reasoning is accurate (or inaccurate) you must (ultimately) appeal to something immaterial.

I would argue that "reason" is a label we apply to a set of processes that brains engage in, and which James Ross calls a simulation of reasoning. The Platonic ideals are also simulations that people use to model abstract concepts. We make models to help us interpret what we observe, and those models include the laws of logic, mathematics, and physics. The universe doesn't obey these laws because they were imposed by a cosmic law-giver. The laws were invented as models of how the universe works. They are descriptive, not prescriptive.


Now I'm just plain curious. Is that what you left with after a thorough reading of the text? Do you think this amounts to a refutation? A refutation of what? What do you think you're addressing? Are you just an assumption-bot? Turing was right after all.

Also, I should be ignoring the red herrings from now on, as there's some serious bait-and-switch here.

So let's back up: You gave up trying to show that babies is really an example of reason arising from non-existent reason, that was supposed to present a counter-argument to Victor's OP, as well as admitting that your referral to quantum vacuums was just irrelevant hand-waving?

@David

Yes, people tend to disagree. That's why we have rational discourse. To decide which side has the better arguments.

If disagreement is a sufficient reason to doubt reason (pun intended), there's hardly much left to hang on to? Where could we ever stop our skepticism to reason itself?

Problem with most modern skeptics, is that they're not nearly skeptic enough.

Unknown said...

@David Brightly: "This is reason to doubt our faith in reason"

Where have I heard that line before? Oh yeah, Sye Ten Bruggencate, who thinks that since reasoning is based on axioms, that he should be able to use... wait for it... the bible as an axiom.

So-called "debates" with him are a complete waste of time, and degenerate to cries of "brain in a vat!". I could just as readily presuppose that his god doesn't exist, and that Bruggencate is a fungus being from the planet Yuggoth. If I pick those as axioms, I can assume they're true, right? :)

B. Prokop said...

"@B. Prokop: Why do you refer to people as "gnus"?"

Gnu is the term that militant anti-theists normally use to refer to themselves. That is why I use it.

Jezu ufam tobie!

Anonymous said...

Suth writes:

It's also very difficult to prove either way, since we don't have an example of nothing to study. At best we can say that something can come from something, and that we don't know of any examples of something that did not come from something.

...and yet, Suth alleges that it's *OUR* views that are inconsistent with science, and consistent with magic.

There's nothing more blinding than just enough intelligence to make you think you know a thing or two!

Anonymous said...

...and to whichever ignoramus atheist brought up machines as an alleged example of reason coming from non-reason, try again: computers are material things that have had reason imbibed in them. They are NOT an example of reason arising from non-reason, or "material things thinking," or anything like that.

If nobody brought that up here, it must have been somewhere else.

Unknown said...

@Daniel Joachim: If you thought the article was irrelevant to the discussion, why link to it? If you think I missed the point, please identify what you think the article was about.

I thought it was arguing that physical beings are incapable of reasoning, and implied that since humans are capable of reasoning, the part of us that reasons must be at least partially immaterial. It explicitly rejected the idea that humans only simulate reasoning, and used the idea that reason is immaterial to defend that point.

I refuted that argument by pointing out that what we refer to as reasoning is a physical process we perform, not an immaterial ideal to be simulated. If you want to claim that reasoning is immaterial, you need to show what part of that process cannot occur physically, instead of assuming humans can do something a machine can't. Perhaps you should read up on neuroscience, so you can see how far we've come in mapping out which parts of the brain perform which cognitive operations.

I guess you're not used to the idea of having to demonstrate the truth of an assertion with actual evidence. Also, I don't accept ideas just because they were invented by respected people, or are thousands of years old, or many people agree with them, or they appear in journals or textbooks. Ideas stand and fall on their own merits. I am free to refute anyone's statement if it doesn't make sense.

Am I the only one who sees that you can't make an argument for the existence of immaterial things if you start with the assumption that they exist? Start with the assumption that they don't exist, and see whether you reach a contradiction, or can find evidence to the contrary.

Turing may have been right. A robot that can simulate reasoning as well as a human should probably be treated like one, with all the same rights and responsibilities. After all, we can't determine whether it's actually reasoning or just simulating it, right?

And yeah, the bit about particles popping into and out of existence was a throwaway remark. I was only pointing out that you should question all assumptions, even obvious-sounding ones such as "nothing comes of nothing". I don't really want to get into a discussion of what "nothing" means. It would be immaterial. :)

Unknown said...

@cl: I agree that the computers we currently have do not reason in the same sense that we reason. They are incapable of abstraction, and merely juggle bits around. Any meaning in those bits is based on our interpretation of them, in much the same way that the marks on a page aren't meaningful unless someone reads them.

Our brains are far more complex than any computers we have been able to manufacture, and are also structured very differently. Even the simplest operations they perform are done in an extraordinarily complex way.

Unknown said...

@cl: "...and yet, Suth alleges that it's *OUR* views that are inconsistent with science, and consistent with magic."

How is *not* making an assumption about a phenomenon we have not observed inconsistent with science, or in any way related to magic?

You can believe in supernatural things and still do science. Just leave those ideas at the door, and employ methodological naturalism. How you deal with the resulting cognitive dissonance is up to you.

Daniel Joachim said...

@Suth

Babies, man. Babies.

Geez, what a big mess to avoid the topic at hand.

Funniest thing is you probably don't realize the blindingly obvious irony in the rest of your rant.

(And yeah. Outside your little land of straw, methodological naturalism is completely superfluous to science. How you deal with the resulting cognitive dissonance is up to you.)

David Brightly said...

Surely there's lots left to hang on to? Day to day reasoning about concrete things such as why the car won't start is highly successful. As it is in mathematics and the physical sciences. There are two big areas where reason runs into trouble. The first is in understanding each other at a personal level. This extends into the humanities and into history and politics. The second is the domain of 'ultimate questions', of philosophy and religion. Reason's ability to generate widely accepted answers runs out of steam. Or, to mix metaphors, the arguments run into the sand. I think there are reasons(!) for this but they only make sense relative to my naturalistic understanding of what reason is, so there is little point in rehearsing them here. Maybe it's worth saying that in these domains we have only Reason itself to guide us, and perhaps personal experiences of a singular nature which aren't open to all. I don't trust Reason that remains untethered to the ground. The history of philosophy surely shows how easily it runs astray. And I can't speak about and evaluate experiences I've not had.

Daniel Joachim said...

I don't trust Reason that remains untethered to the ground

I agree. That's why I'm an Aristotelian and a Scholastic, and why I'm not clear, same as the OP, as to what "naturalistic reason" could even mean. Few people say it clearer than Etienne Gilson.

Key takeaway: Insufficient grounding. Where first principles are not sufficiently established, the rest seem trivial.

“Since, however, it had become clear to me that, technically speaking, the metaphysics of Descartes had largely been a clumsy overhauling of scholastic metaphysics, I decided to learn metaphysics from those who had really known it, namely, those very Schoolmen whom my own professors of philosophy felt the more free to despise as they had never read them. Their study has wholly convinced me, not at all that to philosophize consists in repeating what they have said, but rather that no philosophical progress will ever be possible unless we first learn to know what they knew.

The chaotic condition of contemporary philosophy, with the ensuing moral, social, political, and pedagogical chaos, is not due to any lack of philosophical insight among modern thinkers; it simply follows from the fact that we have lost our way because we have lost the knowledge of some fundamental principles which, since they are true, are the only ones on which, today as well as in Plato’s own day, any philosophical knowledge worthy of the name can possibly be established.

(…)

The great curse of modern philosophy is the almost universally prevailing rebellion against intellectual self-discipline. Where loose thinking obtains, truth cannot possibly be grasped, whence the conclusion naturally follows that there is no truth.”

Ilíon said...

B.Prokop: "Gnu is the term that militant anti-theists normally use to refer to themselves."

The "new atheist" God-haters first tried to brand themselves the "Brights" (*), which is why I decided that Litebtite is a fitting nickname for our most recent Special Snowflake. After that flopped, they settled on "gnu", apparently hoping to rope in the "open source software" folks, or at least ride the coattails of whatever small cashet they have.

(*) the very analogy *they* used to describe their hope to change the meaning of "bright" is how militant sexual perverts have managed to change the basic meaning of "gay"

Ilíon said...

"The great curse of modern philosophy is the almost universally prevailing rebellion against intellectual self-discipline. Where loose thinking obtains, truth cannot possibly be grasped, whence the conclusion naturally follows that there is no truth."

I trace this "post-modernism" to the need to protect Darwinism from rational critical evaluation. The thing is, just as you can't have a little bit of sin, you can't have a little bit of intellectual dishonesty; and so the intellectual dishonesty necessary to protect Darwwinism from rational critical evaluation has spread throughout the sciences and alleged sciences, such that even physicists are chasing after anti-scientific "theories".

Ilíon said...

cl: "...and to whichever ignoramus atheist brought up machines as an alleged example of reason coming from non-reason, try again: computers are material things that have had reason imbibed in them. They are NOT an example of reason arising from non-reason, or "material things thinking," or anything like that."

Actually, even that overstates the case of what a computer is. What a computer *is* is a motorized abacus. And, just as an abacus does not exhibit reason "arising" from non-reason, nor from mere matter, neither does a computer.

Daniel Joachim said...

I trace this "post-modernism" to the need to protect Darwinism from rational critical evaluation. The thing is, just as you can't have a little bit of sin, you can't have a little bit of intellectual dishonesty; and so the intellectual dishonesty necessary to protect Darwwinism from rational critical evaluation has spread throughout the sciences and alleged sciences, such that even physicists are chasing after anti-scientific "theories".

That's hardly fair. Why this constant insistence on intellectual dishonesty? How do you know?

Why attack Darwinism? As long it's referring to evolution qua natural science, it's completely orthodox. It's the accompanying naturalism that seems to be at odds with it. Primarily, because there may be no such thing as a thing, and hence nothing to evolve from a real thing to another real thing.

Too complex to elaborate upon here, but you'd probably enjoy this. Endorsed by Charles Taylor, William Desmond and Slavoj Zizek alike.
http://www.youtube.com/watch?v=w3k8kMabvjw

B. Prokop said...

The fatal flaw in all arguments that run along the lines of "We can observe the physical processes within the brain that accompany thought, and therefore those physical processes are all there is" is that one could make the same argument for the eye or the ear (or for any other sense organ). It would look something like this: "We can observe (pun intended) how the eye processes light, therefore there is no such thing as light."

Just because we can observe how the brain processes thought does not mean that there is no such thing as thought.

And please, please don't respond with "But we can show that light actually exists - not so for thought." That is false on its very face. The fact (brought up by Wizard himself) that we experience periods of unconsciousness demonstrates the existence of thought (by its ability to be either present or not present). In the same manner, there is both light and darkness.

Saint Hildegard von Bingen, Pray for us!

Anonymous said...

How is *not* making an assumption about a phenomenon we have not observed inconsistent with science, or in any way related to magic?

Do you really not see what I'm getting at here? I mean, come on.. I know you're the smart, rational atheist, and I'm just the dumb-dumb faith head, but.. it's all right there. Your prior words,

At best we can say that something can come from something, and that we don't know of any examples of something that did not come from something.

..how *don't* those words commit you to at least a tentative Aristotleian theism? You just echoed the basic tenets, and you're right: all the physical evidence we have says that something can come from something, and that we don't know of any examples of something that did not come from something. So there you have it: the totality of "physical" evidence supports our views. Everything did come from something, not nothing.

But go ahead and keep talking down your snoot like you're better than everybody else here and see what that does for ya in terms of critical thinking and opening new mental doors.

Unknown said...

@Ilíon: "What a computer *is* is a motorized abacus"

A computer is anything that takes in data, processes it in some organized way, and produces data derived from it. A brain is a type of computer. It takes in sensory input, interprets it, stores a representation of it, performs various operations on it, and produces output in the form of nerve impulses.

The word "computer" used to refer to a person who performed calculations. The modern usage was originally an analogy.

You're against atheists ("God-haters"/"God-deniers"), philosophical materialists ("naturalists"), homosexuals ("militant sexual perverts"), and evolutionary biologists ("Darwinists"). I find it odd that you complain about a lack of intellectual integrity in others when your own position appears to be rooted in emotion.

I also find it ironic. I abandoned religion primarily because I valued intellectual integrity more than emotional security.

B. Prokop said...

"I abandoned religion primarily because I valued intellectual integrity more than emotional security."

Very telling words there. There is no need for intellectual integrity and emotional security to be at odds. In fact, there is great reason to regard them as mutually supporting - you can't even have one without the other.

The problem with most "hard core" atheists that I have come across (as opposed to the sadly far more common "lazy" variety, who profess atheism out of apathy) is that they quite unnecessarily deny half (or more) of their humanity, restricting themselves to a narrowly defined, rigidly circumscribed subset of all the ways one can interact with and learn about the world around us.

I have likened the situation to a handyman who insists on trying to do every job with a single tool. (The atheists loves to label this tool "reason" or "logic" or "science", etc.) But not everything is a nail, and sometimes you need to use other tools than just a hammer.

The believer, on the other hand, makes use of the entire toolbox. It is possible (in fact, mandatory) to sometimes use art, music, philosophy, history, literature, folk wisdom, tradition, emotion, family, liturgy, life experience, suffering, prayer, human relations... all kinds of things, to build up a coherent picture of objective reality. The man who artificially restricts himself to "reason" alone can (and usually does) end up arguing in a circle. He can't see outside of the box he has imprisoned himself in, because he won't look!

(The scare quotes are there because what the atheist regards as "reason" is in reality, because of its use in isolation, "un-reason".)

Ave, Maria!

Ilíon said...

"I have likened the situation to a handyman who insists on trying to do every job with a single tool. (The atheists loves to label this tool "reason" or "logic" or "science", etc.) But not everything is a nail, and sometimes you need to use other tools than just a hammer."

Myself, I prefer to turn the saying around: all the so-called atheists have is a nail, and so they insist that The One True Tool is a hammer.

Unknown said...

@B. Prokop: "There is no need for intellectual integrity and emotional security to be at odds.

I agree. I just meant that I had to give up the false emotional security supplied by religion. It had been an important part of my life and my sense of identity for decades.

I used to believe that atheism would imply nihilism. It's one reason I was afraid to question religious dogma. It turns out that assumption wasn't true at all. I don't wallow in despair because we are insignificant specks in an indifferent, meaningless universe. Actually most aspects of my life didn't change much.

I had to re-evaluate my views of morality and their basis. The result was that I stopped trying to despise people, such as homosexuals, whom the church had told me were evil. I rejected the religious idea of sin as something that you can inherit, and that can only be removed by substitutionary atonement. I stopped automatically respecting people who claim to speak for undetectable beings.

Overall it's been a positive change. I am now free to think about or question anything I choose.

Unknown said...

@B. Prokop: "Just because we can observe how the brain processes thought does not mean that there is no such thing as thought."

I never claimed that thought doesn't exist. I just don't think it requires anything immaterial. It appears to be an emergent property of physical processes.

B. Prokop said...

"I used to believe that atheism would imply nihilism."

It does. It implied it when you believed it, and it still does today. Your change of mind did not change the reality that atheism = nihilism.

"I don't wallow in despair"

No need for you to so so. Doesn't change the fact that atheism = despair.

"we are insignificant specks in an indifferent, meaningless universe"

For an atheist, that is Ground Truth. There's your nihilism that you were denying just three sentences ago.

"I stopped trying to despise people, such as homosexuals, whom the church had told me were evil."

Wow. I must have been absent that day. I never started trying to despise people. No need for me to stop.

"I stopped automatically respecting people who claim to speak for undetectable beings."

Bravo! If that's what you were doing, then it's probably why you lost your faith. No one ought to "automatically" trust anyone on anything. "Test** everything, and hold fast only to what is good." (1 Thessalonians 5:21)

** Alternate translation: "prove"

"I never claimed that thought doesn't exist."

Glad to hear that. Many atheists on this site have insisted it doesn't exist.

" It appears to be an emergent property of physical processes."

More gobbledegook. Semantically null.

Jezu ufam tobie!

Anonymous said...

Bob, that entire comment that ended Ave Maria was a true gem. Salud! Some great writing there, a real hammer.

Anonymous said...

I found this exchange interesting:

"I stopped trying to despise people, such as homosexuals, whom the church had told me were evil." - Suth

Wow. I must have been absent that day. I never started trying to despise people. No need for me to stop. - Prokop

...the religious-turned-atheist are seriously interesting to me. I dug up this comment from a few years back, I left it at CSA but I think it's relevant here, I'll explain why after:

…I think deconverted fundamentalists and evangelicals tend to make the most dangerous atheists. Often, the mental weaknesses that led them to dogmatic ways of thinking in the first place persist. These traits then carry over into their newly-embraced “skepticism,” the weaknesses again take the helm, yet, this time, they are actually far worse off because a thin veneer of rationalism masks their dogmatic and irrational tendencies and puffs many up with a false sense of confidence. Consequently, many mistakenly believe that their change of psychological allegiance solved the problem. They’re often less likely to see it, because they still have that “in the tribe” mentality, only now, they fancy themselves in the right tribe. I cannot overemphasize the threat this phenomenon poses to critical thinking and pursuit of truth. Trading one’s cross for a scarlet A accomplishes nothing unless the old habits are shed.

...to me, Suth didn't shed that "despising the other tribe" part of the archetype. Rather, the contempt and the intellectual arrogance that inevitably comes along with thinking one has rational and/or moral highground naturally flipped from gay people to religious people - and you can read it here, in the comments. This "despising the other tribe" mentality is subtle and toned down a bit probably no doubt because of the newfound moral / intellectual superiority, but it's there.

Just another product of polarized culture wars I guess.

Secular Outpost said...

I had asked:

Why do you think [nihilism, amorality, and despair] are the logical implications of atheism?

planks length replied:

Unfortunately, for no original reasons; none that haven't been laid out a thousand times before already.

Basically, without individual immortality, in the long run everything is totally, completely, and utterly meaningless. The instant we die, it is quite literally as though everything we ever did, everything we ever experienced, everything period, might as well never have happened. Thus the nihilism and despair.


First, sorry for the delay in responding. I somehow managed to miss your reply when it was first posted.

Second, I think I agree with you that the absence of personal immortality seems to entail the lack of eternal significance. I don't understand how you move from "lack of eternal significance" to nihilism. It seems to me that is a crucial step, one I don't find a supporting argument for in your comments.

Third, along similar lines, I don't understand how you move from "my actions lack eternal significance" to despair. It's as if you want to say, "It's an objective fact that you should subjectively feel despair." It seems to me that despair is an emotion which, by definition, is a subjective state. What non-question-begging reason is there to think, "If person P thinks his actions lack eternal significance, then person P should feel despair"?

Next. In the absence of an almighty and everlasting God, all "morality" is just opinion, no matter how you gussy it up. In other words: amorality.

Why think that?

Anonymous said...

PL / Jeff

Sorry for butting in but I find your conversation interesting...

Next. In the absence of an almighty and everlasting God, all "morality" is just opinion, no matter how you gussy it up. In other words: amorality. -PL

Why think that?-Jeff

Because as humans our reasoning is inherently inaccurate and flawed, and we are all on equal footing with one another in that regard. Who's to say what morality is without appeal to an authority larger than ourselves?

Jeff, you said earlier that "nobody has shown a logical contradiction between objective morality and atheism" ... (or something to that effect), well here it is, staring you right in the face: sans said authority, ALL our moral banter is dependent on the subjective states of humans.

Done deal.

Unknown said...

@cl: "Everything did come from something, not nothing."

Do you also believe that all swans are white? Europeans used to, until they visited Australia. The totality of the available physical evidence supported their view.

I'm not assuming that something can came from nothing, but I'm not assuming that it can't either. I have no data about nothingness on which to base any such claim.

I'm open to the idea that thought might be something immaterial that just happens to be closely correlated with brain activity. I simply have no reason to make that assumption.

Anonymous said...

I have no data about nothingness on which to base any such claim.

Uh, yeah, but you have an ABUNDANCE of data about how somethingness apparently requires somethingness, so you might want to look both ways.

Unknown said...

@cl: "I think deconverted fundamentalists and evangelicals tend to make the most dangerous atheists"

Are you really that afraid of people who don't share your point of view? What do you think I'm going to do, disagree with you some more?

Anonymous said...

Are you really that afraid of people who don't share your point of view?

Not at all. In cherrypicking the sentence you needed for ammo, you missed the entire point. The "danger" I allege the deconvert poses is not to theists like me who are completely intellectually satisfied in their beliefs. Rather, the "danger" is to the practice of critical thinking in general, and more specifically to those whose eyes aren't yet trained to see that all you did was switch your cross to a Scarlet A; that all those traits still persist. Like I said, the thin veneer.

planks length said...

Jeffery,

Never thought of despair as necessarily being an emotion, but rather as a descriptive of circumstances, similar to "The situation was hopeless." No one need feel the absence of hope for there to objectively be none. If you object to the word despair, then simply substitute hopelessness and re-read the comment.

Why think that?

I think you're putting the cart before the horse. The real question ought to be "Why not?" The burden of proof is on the person to show why his opinion is somehow objective. And without a perfect standard (almost by definition, God) by which to measure, there is no defense against everything being mere subjectivity.

Unknown said...

@planks length: "And without a perfect standard (almost by definition, God) by which to measure, there is no defense against everything being mere subjectivity."

Adding a god doesn't solve the problem of objectivity. It just shifts the subjectivity to the god. I expect you are already familiar with the Euthyphro dilemma.

Anonymous said...

Suth,

Adding a god doesn't solve the problem of objectivity.

You're flat-out wrong here, at least in the context of Jeff's definition of "objectivity," which reduces to "not subject to the mental states of humans."

B. Prokop said...

"I expect you are already familiar with the Euthyphro dilemma."

If you'd kept up with this site, you would know that the Euthyphro dilemma was blown out of the water not that long ago. Simply replace the "because" with an "and" and presto! No dilemma. Done. Dusted. Move along - nothing to see here.

Jezu ufam tobie!

B. Prokop said...

RE: My own comment of May 15, 2015 9:14 AM.

Long time devotees of Dangerous Idea know well that I scoff each time some atheist or other accuses believers of "living in a box" whereas in reality, it is they who inhabit boxes - and suffocating ones at that. Self imprisoned within stifling walls of a woodenly rigid materialism that allows for no faintest breath of fresh air from the infinite realms that surround, indeed that cradle, this speck of a universe that we inhabit, the atheist limits himself by his own choice to the crumbs of reality that fall his way off the table of "empirical evidence".

There's no wonder why I have no bad feelings whatsoever toward atheists - I pity them. I reserve my hatred (let's not mince words and call a spade a spade here - the word "hatred" is exact and appropriate) for the damnable doctrines of atheism itself, not for those unfortunates who have sorrowfully fallen under its wheels.

Where was I? Oh yeah... Kudos to Ilion for pointing out THIS LINK, which blows wide open the materialist's box, allowing clean, invigorating air from the Blessed Realms to flow over the fetid, stagnant mere of atheism.

Wonderful stuff!

Jezu ufam tobie!

Secular Outpost said...

Sorry for butting in but I find your conversation interesting...

No problem. Sorry for ignoring your butting in, but it's a 1:1 conversation on a public forum. ;-)

Unknown said...

@B. Prokop: "Simply replace the "because" with an "and" and presto! No dilemma."

That's just a way of using word games to ignore the question. You can't resolve the dilemma, so you refuse to address it.

I will rephrase the question more directly.

Is God the source of morality?

If the answer is yes, morality is both arbitary and subjective. If God decreed that it was moral to murder children, it would actually be moral. He could make up silly rules, such as wearing tassels on your clothes, not eating shrimp, or only turning on your oven on weekdays, and they would become moral laws.

If the answer is no, God is irrelevant to morality. It should be possible to determine what is moral without reference to God, whether morality is a set of absolute universal principles, mere opinion, or anything in between.

I don't believe that any god exists to act as a source of morality, so we are obliged to figure out what is moral for ourselves. Even if a god did exist, accepting its moral tenets without first independently evaluating whether they are moral would mean that you are not acting as a moral agent. You would be replacing morality with blind obedience.

Unknown said...

@B. Prokop: "Doesn't change the fact that atheism = despair."

That sounds like an appeal to consequences. You choose to believe the claim that a god exists because you don't like the implications if the claim is false. You think you would despair, because you believe life would have no value without a god and an afterlife.

Avoiding a question because you are afraid of the answer is intellectually dishonest.

I didn't choose to become an atheist. I chose to be skeptical of claims that appear to have no basis in fact. I recognized that I had built mental barriers around my religious beliefs to prevent myself from questioning them, so I tore the barriers down. I honestly considered the idea that there might not be a god, and thought about the implications. How different would the world be if there were no god, or anything else supernatural?

The answer is that it would be no different. The god I had been taught to believe in hides its existence, and souls are "immaterial". Nature seems completely indifferent to us. Religious rituals and prayers have no discernable effect. The promise of Heaven and the threat of Hell both take place after death, and no one can verify that either one exists. Miracles are all anecdotal or might have happened anyway. So-called "religious experiences" are entirely subjective, and can be induced in other ways. People are carefully trained never to doubt or question anything the clergy tell them, and are almost always taught to believe religious claims for emotional reasons before they've developed the ability to analyze those claims critically. More than 30,000 sects of Christianity base their beliefs on the same "divinely inspired" book, but they all disagree about what it means, and the book itself is riddled with inaccuracies and contradictions.

It has all the hallmarks of a scam, and a very successful and profitable one too. They've even convinced secular governments to make them tax-exempt, with no requirement to disclose their revenue, assets, or spending. The real threat of atheism is that takes financial and political power away from churches.

You can go on believing in religion if you want. I don't buy it anymore.

And, no, I'm not a conspiracy theorist. As far as I can tell the vast majority of religious people, including the clergy, actually believe in it, as did I. They think what their church does is right and good and holy, and that they're spreading the truth to save people from eternal torment. I have no bad feelings toward them, but sometimes I pity them, to coin a phrase. They're so wrapped up in their religion they think life would be meaningless without it.

Unknown said...

@cl: "You're flat-out wrong here, at least in the context of Jeff's definition of 'objectivity,' which reduces to 'not subject to the mental states of humans.'

What a conveniently human-centric, and hence subjective, definition of subjectivity that is.

Do you think that ideas can't be subjective just because they didn't originate from humans? Is a dog's opinion of a squirrel objective?

If the definition extends beyond humans to other beings with mental states, why shouldn't it apply to a god? Doesn't your god have a mental state? Or is your god mindless? How can you know either way?

It sounds to me like a case of special pleading.

Unknown said...

@Daniel Joachim: I've been thinking a bit more about James Ross's article on immateriality, and I think I've found a refutation for another of the points he makes. I didn't notice a problem with it before, but it is crucial to his argument.

The article asserts (with some justification) that every physical machine is inherently indeterminate. That is, there is no way to be certain that it performs a given function, e.g. multiplication, for every possible set of inputs, and not some other function that gives a different result in some set of cases.

I'm not sure this is true. I don't have a formal disproof, but I think I can illustrate the problem with an example.

Suppose I make a calculator that performs a single operation on a single input value. It multiplies whatever numeric argument you pass it by zero. The design of this calculator is very simple. I take a piece of paper and write the numeral 0 on it. Anytime I want to multiply a number by zero, I take out the piece of paper and look at it. I don't even need to enter the argument, because the calculator predicts the correct answer every time.

Is my calculator determinate? Yes. It produces the correct answer for every possible input, exactly the same as if it had reasoned the answer. It perfectly simulates the formal operation of multiplying by zero. It will continue to do so as long as it remains intact.

I suppose one could argue that the paper might be damaged, and then not show the right result. I could also argue that a human's brain might be damaged so that its owner is no longer able to multiply, even if they thought they still could. Breaking the machine does not show that it is indeterminate.

That reminds me of another point I was wondering about. The article seems to assume that because humans are capable of actually reasoning, that humans always reason correctly. This is obviously untrue; people make mathematical errors and draw false conclusions all the time. In what way is human reason any more determinate than a simulation of reason produced by a physical system? Indeed, humans seem more prone to error than many existing physical systems.

B. Prokop said...

"You think you would despair, because you believe life would have no value without a [G]od and an afterlife."

Wrong once again. I don't "think" so, I know it.

"I didn't choose to become an atheist."

Sorry, but Yes, you did.

Wow. Your next paragraph is a real pile there. Let's shovel through it.

"The [G]od I had been taught to believe in hides [His] existence"

No, He does not. "For what can be known about God is plain, because ever since the creation of the world his invisible nature, namely, his eternal power and deity, has been clearly perceived in the things that have been made."


"souls are "immaterial""

Yes and no. Human beings are a composite creature, composed of body and soul. Their separation at death is unnatural and a consequence of sin. In any case, they will be reunited at the Last Day.

"Nature seems completely indifferent to us."

Really? It doesn't seem so to me.

"Religious rituals and prayers have no discernable effect."

Amazing statement. You really ought to read the diary of Saint Faustina, and then try to say that with a straight face afterwards.

Just as an example, I myself started praying the Rosary daily some time ago, and the effects have been immediate, discernible, and quite profound.

"The promise of Heaven and the threat of Hell both take place after death"

Wrong once again. Heaven begins here and now. As Dorothy Day wrote, "All the way to Heaven IS Heaven, because Jesus said, 'I am the Way.'" And have you never heard the phrase "a living hell"?

"No one can verify that either one exists."

I don't know about that. I know that I can't, but I wouldn't presume to speak for everyone.

"Miracles are all anecdotal or might have happened anyway."

Everything that you do not personally experience is "anecdotal" - World War II is anecdotal. So what?

"So-called "religious experiences" are entirely subjective, and can be induced in other ways."

Read what I posted above about the eye. Irrelevant observation.

"People are carefully trained never to doubt or question anything the clergy tell them"

Hmmm... I wasn't.

"More than 30,000 sects of Christianity base their beliefs on the same "divinely inspired" book, but they all disagree about what it means"

We're in agreement here, but we wouldn't have this problem had it not been for the Protestant revolt (a.k.a., the "Reformation"). It's one huge reason why I am a Catholic. Problem solved.

"the book itself is riddled with inaccuracies and contradictions"

Only for people who do not understand how to read.

Boy, if that's why you chose to become an atheist, then you wuz robbed!

(I'll get to your question about the Euthyphro non-dilemma later today. I'm off to Mass, and I generally spend a good 3-5 hours on Sundays at church, so don't expect an answer soon.

Jezu ufam tobie!

Anonymous said...

Suth,

Could you provide a concrete example?

Sure, once I have some evidence that you debate in good faith. As it stands, I've left a solid handful of rebuttals to comments you've made, and I'm more interested in finishing those conversations than starting another one.

Unknown said...
This comment has been removed by the author.
Anonymous said...

Lowder,

"it's a 1:1 conversation on a public forum"

Ah, now that's an easy way to relieve yourself of the duty of a rejoinder :) haha. Seriously though, Jeff, when you get the time, I would like to hear your response because it seems wrong what you said about nobody showing a contradiction between objectivity morality and atheism. I think you need to amend your claim(s).

Suth,

....and, once again I miss your comment back up the thread. Sorry Suth.

What a conveniently human-centric, and hence subjective, definition of subjectivity that is.

This reeks of sophistry. What, now you're going to cry "subjective" to every definition offered? Suth, it's impossible to provide a definition that isn't subjective. I didn't come up with Jeff's definition. Do you have a better one?

Do you think that ideas can't be subjective just because they didn't originate from humans?

Not sure what you're asking there, you seem confused...

Is a dog's opinion of a squirrel objective?

Why assume that dogs can form opinions when we have no evidence to support that assertion?

Honestly Suth, if you want to have a good discussion about this, you need to provide *precise* definitions for your terms. If you do that I'll talk more. Until then, forget it, this is just an exercise in talking past one another.

Define "objective" and "subjective"

Unknown said...

@B. Prokop:

"Wrong once again. I don't 'think' so, I know it."

How do you know? You aren't in that situation.

Romans 1:9

That's a teleological argument, also known as an argument from ignorance. You don't know how the universe began, why it exists, how life began, how consciousness works, etc., so you assume that any explanation is better than not knowing. You're using the label "God" as a placeholder for "I don't know". It's similar to the way the writers of the T.V. show Xena explained away plot elements: "A wizard did it".

The thing that made the things for which there is no known maker

"Human beings are a composite creature, composed of body and soul."

That's the claim. I have no reason to accept it.

"Nature seems completely indifferent to us."

"Really? It doesn't seem so to me."

So natural disasters and diseases only harm evil people? That's the sort of hateful thing Pat Robertson says: Don't help the people of Haiti recover from an earthquake, because they brought it on themselves, as did those godless Japanese.

"You really ought to read the diary of Saint Faustina"

More anecdotes. Did Saint Faustina perform religious rituals under lab conditions? Were the findings consistently reproducible by independent researchers? Did the papers about it pass scientific peer review?

"I myself started praying the Rosary daily some time ago, and the effects have been immediate, discernible, and quite profound."

I should have said "no discernable effect beyond placebo". Prayer can have an effect on the mental state of the one praying, as can non-religious meditiation.

Unknown said...

@B. Prokop:

"Wrong once again. Heaven begins here and now."

Equivocation.

"No one can verify that either one exists."

"I don't know about that. I know that I can't, but I wouldn't presume to speak for everyone."

Okay, so how would someone go about demonstrating that Heaven or Hell exists? If you can't verify the truth of a claim, even in principle, there's no reason to accept it.

"Everything that you do not personally experience is "anecdotal" - World War II is anecdotal. So what?"

The evidence for World War II is far better than that for any miracle, and it's not an extraordinary claim.

"Read what I posted above about the eye. Irrelevant observation."

My point was that you can't use those subjective experiences as evidence of anything supernatural.

"People are carefully trained never to doubt or question anything the clergy tell them"

"Hmmm... I wasn't."

I guess you didn't go to a religious school. My statement is true for millions of children every year. They are trained to treat everything about religion with automatic respect. Some churches go further, and teach children that they are worthless and will be tortured forever unless they accept Jesus. They make up a fake disease so they can sell a fake cure.

"It's one huge reason why I am a Catholic. Problem solved."

You don't find it even a little disturbing that people find thousands of entirely different and contradictory interpretations of what is supposed to have been inspired by a single, perfect being? Shouldn't it unify people instead of dividing them?

"the book itself is riddled with inaccuracies and contradictions"

"Only for people who do not understand how to read."

Or for people who know how to question what they read. For most people, religion is like a software license agreement. They gloss over the details and pick "I accept".

"I'm off to Mass, and I generally spend a good 3-5 hours on Sundays at church"

If your religious beliefs are true and obvious, why do they need to be reinforced every week?

B. Prokop said...

Oh, my. This is just way too much fun. There's got to be something sinful about it!

"That's a teleological argument, also known as an argument from ignorance."

No, it's the exact opposite. I utterly reject the foolish (and probably atheist-invented) notion of a "God of the Gaps". My God is a "God of the Filled In Spaces". To quote myself, "The more we learn about the universe, the more we see the astonishing detail and complexity of its structure, the more we discover its remarkable adherence to regularity and law, the more we appreciate its sheer scale... the more opportunity there is to appreciate the Mind behind it all. The Heavens do indeed proclaim the Glory." (Observing the Nearest Stars, page 98)

"That's the claim. I have no reason to accept it."

Nobody asked you to. What you think does not change what is.

"So natural disasters and diseases only harm evil people? That's the sort of hateful thing Pat Robertson says"

You apparently pooh-pooh The Bible without having a clue about what it actually says. "[God] makes his sun rise on the evil and on the good, and sends rain on the just and on the unjust." or this "Those eighteen upon whom the tower in Siloam fell and killed them, do you think that they were worse offenders than all the others who dwelt in Jerusalem? I tell you, No."

"Equivocation."

Hah! Caught you in that one. Atheists are forever blathering about wanting "here and now" evidence, but give it to them, and they call it equivocation! (What's that I often hear about on this site concerning "Intellectual Dishonesty"?)

"and it's not an extraordinary claim"

I regard the infinitely recycled statement "Extraordinary claims demand extraordinary evidence" to be among the most stupid things ever said by anyone, ever. (a redundantly made claim demands a redundant response)

"My point was that you can't use those subjective experiences as evidence of anything supernatural."

And my point is that you cannot use an understanding of how the body processes religious experiences to be evidence against their supernatural origin.

I guess you didn't go to a religious school."

I went to CCD; does that count? In any case, you're changing the goalposts. Above you wrote, "People are carefully trained never to doubt or question anything the clergy tell them." Now you're changing that to "trained to treat everything about religion with automatic respect." I have zero objection to the latter. Yes, people should treat everything about religion with automatic respect. Not acceptance or belief, mind you, but yes - with respect.

"You don't find it even a little disturbing that people find thousands of entirely different and contradictory interpretations of what is supposed to have been inspired by a single, perfect being?"

No. I'll start worrying about it when atheists start worrying about how nonbelievers can embrace adherents of both Marxist Communism and Ayn Randian Objectivism.

Bonus Point: God is not "a being" - He is Being. Big difference.

"Or for people who know how to question what they read."

Once again, you betray your seemingly bottomless ignorance of how to read. You cannot read The Bible without questioning everything. Good Lord, man! Have you never read Augustine, Aquinas, Charles Williams, Daniel Berrigan, George MacDonald, Maximilian Kolbe? Questioners all.

"The Heavens declare..."

B. Prokop said...

""I'm off to Mass, and I generally spend a good 3-5 hours on Sundays at church"

If your religious beliefs are true and obvious, why do they need to be reinforced every week?
"

Forgot to respond to this one!

You need to change "every week" to "every day" - I go to Mass daily. Sundays are just longer**. Do you not eat every day? Or drink? Do you not breath continually?

** Today, for instance, started with private prayer, followed by the Exposition of the Blessed Sacrament, followed by the Litany of the Saints (sung in Polish, by the way), followed by Mass itself (also in Polish), followed by a Baptism, followed by a recitation of the Chaplet of the Divine Mercy, followed by a coffee and cake social in the basement (which I skipped today).

Jezu ufam tobie!

B. Prokop said...

And finally, to the Euthyphro non-dilemma:

That's just a way of using word games to ignore the question. You can't resolve the dilemma, so you refuse to address it.

No, the so-called dilemma is itself just playing games with words, on the order of “Can God create a rock so heavy that He Himself cannot lift it?” That’s what your “dilemma” resolves to. At the end of the day, there’s no dilemma to resolve, just as one cannot answer a nonsensical question like the one in my first sentence. If I “fail” to answer that question, are you going to say I’m refusing to address it (when it is manifestly nonsense)?

Goodness (or morality) is an inherent property of God – one of the “all” attributes (all-good, all-knowing, all-present, all-powerful, eternal, etc., etc.) You say that if God is the source of morality, then morality is both arbitrary and subjective. How so? Is knowledge, by the same token, arbitrary? Or presence? Or power? Existence itself? See the types of rabbit holes you dive into once you play fast and loose with words?

Jezu ufam tobie!

grodrigues said...

@Wizard Suth:

Just in case you are wondering, there is not a single cogent point in your comments about J. Ross's argument, besides showing that you do not understand it. Not one. Not really surprising, but always worth pointing out.

Just one example.

"The article seems to assume that because humans are capable of actually reasoning, that humans always reason correctly."

Nowhere does James Ross make that assumption; nowhere. Since it is so obvious that people make mistakes all the time, even quite elementary ones, how could it have escaped the attention of Ross? A case of projection maybe?

Even more:

"This is obviously untrue; people make mathematical errors and draw false conclusions all the time. In what way is human reason any more determinate than a simulation of reason produced by a physical system? Indeed, humans seem more prone to error than many existing physical systems."

Which just goes to say that you really do not grasp Ross's argument. For the fact that people do make mistakes only vindicates his argument, as he himself explains in section III, "Retreat from People". And you say you have read the argument? Yeah, right.

Ilíon said...

litebright: "If your religious beliefs are true and obvious, why do they need to be reinforced every week?"

B.Prokop: "Forgot to respond to this one!

You need to change "every week" to "every day" - I go to Mass daily. Sundays are just longer**. Do you not eat every day? Or drink? Do you not breath continually?
"

Or, to put it into a couple of Biblically-based analogies that just perhaps the willfully dense may comprehend --
You told you wife you loved her once. Can you believe that she expects to hear it again ... and again!

Your wife "made love" to you once. I can't believe that you expect her to demonstrate, yet again, that she loves you!


Christian worship is about loving God ... and learning to love one another.

Ilíon said...

LiteBright: "The article seems to assume that because humans are capable of actually reasoning, that humans always reason correctly."

grodrigues: "Nowhere does James Ross make that assumption; nowhere. Since it is so obvious that people make mistakes all the time, even quite elementary ones, how could it have escaped the attention of Ross? A case of projection maybe?"

I haven't read the article, so I will take grodrigues's word that "Nowhere does James Ross make that assumption; nowhere." And, after all, in the short time that LiteBrite has graced us with his presence, he has displayed a stunning, and familiar, inability to comprehend what he pretends to critique.

LiteBright: "The article seems to assume that because humans are capable of actually reasoning ..."

PREDICTION: if he hangs around long enough, at some point LiteBright will find it convenient to pretend that belief of the truth that human being *can* reason is an "unwarranted assumption", and will demand "proof" of it.

LiteBright: "This is obviously untrue; people make mathematical errors and draw false conclusions all the time. In what way is human reason any more determinate than a simulation of reason produced by a physical system? Indeed, humans seem more prone to error than many existing physical systems."

Now there is a shocker -- "a simulation of reason produced by a physical system" -- a materialist shill implicitly acknowlegding that "physical system" do not actually engage in reasoning. He'll need to walk that back at some poinst (especially as I have drawn the reader's attention to it).

LiteBright: "This is obviously untrue; people make mathematical errors and draw false conclusions all the time. In what way is human reason any more determinate than a simulation of reason produced by a physical system?"

Human reasoning is *not* "determinate" ... which is one important proof of the truth that *no* "physical system" will ever be a mind. It is precisely because we are free -- you know, that important fact about us that materialists must deny, and constantly seek to conflate with "randomness", precisely because it can't be fitted into their false metaphysics, which they're not about to drop for so piddling a reason as it being false to reality -- that we *can* reason.

LiteBright: "... people make mathematical errors and draw false conclusions all the time. ... Indeed, humans seem more prone to error than many existing physical systems."

Yes, people make errors all the time. People *also* recognize errors as errors, and correct them, all the time; both the errors of other persons, and their own. No "physical system" will ever be able to do that; which fact is yet another important proof of the truth that *no* "physical system" will ever be a mind.

David Brightly said...

Hello Daniel,

I have to agree that Aristotelianism and its medieval developments are well grounded and worked out systems of thought. The problem, in my view, is that they are grounded in common sense. Aristotelianism is a theory in and of the manifest image, to use Sellars's term. Aristotle and the medievals can be forgiven for not knowing what we have learned about the world since 1600, which in many ways undermines common sense. The question then becomes whether the manifest image can assimilate the scientific image, or whether the scientific image can assimilate the manifest image, or perhaps neither, and a fresh start is needed. My education lay outside the Aristotelian tradition and I'm not as familiar as I'd like to be with the extent to which Aristotelians see this as an issue, and what efforts are being made to tackle it. Those of us who see the scientific image as primary are well aware of the problems we face in accounting for the manifest image. They have been well-rehearsed in these pages, I am sure. Do Aristotelians think that they face no challenges at all?

planks length said...
This comment has been removed by the author.
B. Prokop said...

"If you claim that reason cannot arise from unreason, or that material beings are incapable of reasoning, you need to support those claims with physical evidence."

I'm aware that this comment by "wizard" is about a week old, but I just now read it for the first time.

Wizard, what you're asking for is like asking "how much does the number 12 weigh?" or "how big is red?" If it is true that reason cannot arise from material processes alone, but rather requires some sort of causation outside the physical universe, then it is absurd from the get-go to demand "physical evidence" for a non-physical agent, and even more absurd to reject the possibility of such solely because it does not fit your artificially limited preconditions.

You're like the drunk who's dropped his car keys somewhere out in the darkness, but insists on searching for them under a lamppost because "the light's better here."

"In the beginning..."

Anonymous said...

David Brightly: The problem, in my view, is that they are grounded in common sense. [...] what we have learned about the world since 1600, which in many ways undermines common sense.

Well, no, and no. (In my experience, claims about what the mediaevals couldn't have known are generally inversely proportional to knowledge of what they actually said.)

Do Aristotelians think that they face no challenges at all?

Well, new discoveries in physics may pose challenges for physicists, Aristotelian or otherwise; but fundamentals like "there are things that exist" or "some things change" are not going to, er, change, any more than new mathematics is going to undermine Pythagoras's Theorem. The biggest challenge those in the Aristotelian tradition face is getting people to understand their actual views instead of crude caricatures.

David Brightly said...

Mr G,
Does the extent of my knowledge of medieval thought (admittedly quite limited, so you can consider your experience now 1001-fold!) have any relevance to how much medieval thinkers knew about the world? The claim is that much more has now been discovered, and some of it is rather undermining of even present-day common sense, which has moved on somewhat since the 13th century, and hence is undermining of Aristotelianism.

For example, we who start in the scientific image, which at its most basic level operates without the notion of 'cause', have an uphill struggle to explain how (efficient) cause arises in the manifest image. Likewise, those who start within the manifest image, and make cause fundamental, have to explain how the scientific image gets by without it. Do present-day thinkers in the Aristotelian tradition see this as a problem, and are they tackling it? I would like to read them.

I would have thought that the history of ideas teaches us that little can be set in stone. Arguably, the work of Bolyai, Lobachevski, and Gauss did undermine Pythagoras, at least until Euclidean geometry was reclaimed and reset in a newer, wider context.

grodrigues said...

@David Brightly:

"Likewise, those who start within the manifest image, and make cause fundamental, have to explain how the scientific image gets by without it."

The "scientific image" does not get by without it. At all. What does happen is that in the *mathematical formulation* of the physical theories, words like "cause", "substance" and "essence" do no do any work. Nor could they, because these are precisely the aspects of reality that cannot be captured by the mathematical formalisms. But that does not mean that these, or their functional equivalents, do not play any role. Of course they do, because scientific theories aim at give a rational understanding of the world, not merely a mathematical description of it useful for prediction purposes.

And *if* you take the latter to be the fundamental job of Science, that is, you take a purely instrumental perspective, then what is so extraordinary that scientific theories do not feature things like "cause", "substance", etc.? They also do not feature things like me, you, Love or the poetry of Wallace Stevens, but that thereby does not cause us to think that the latter do not exist or are less real.

Contrapositively, is everything that does play a role in the physical theory real? There are many things in the physical theories that are merely theoretical artifacts (or at least arguably so). Are the Hilbert state spaces of quantum mechanics real or not? The mathematical description of QM employs them, so by your logic it seems they have independent, extra-mental existence. Now that is a truly fantastical claim. Maybe someone will retort that one can rewrite everything in ZFC or finite order arithmetic or some such. Well, are then sets or sequences of natural numbers real?

The only way your claim would have any weight would be if we already knew that the mathematical descriptions as provided by the scientific theories are exhaustive of reality. But merely assuming that is question begging and is part of the dispute itself.

"Arguably, the work of Bolyai, Lobachevski, and Gauss did undermine Pythagoras, at least until Euclidean geometry was reclaimed and reset in a newer, wider context."

But the fact is that it did not undermine Euclidean geometry; what it did undermine was the belief that there was, or could be (*), no other geometry other than the Euclidean, which is a different thing. This is no different in kind from the belief that all swans were white until a black swan was found.

(*) this is of course, an anachronistic way of putting things. Even in the long, drawn out discussions about the parallel postulate, the issues were not put in these (modern) terms.

Anonymous said...

David Brightly: The claim is that much more has now been discovered, and some of it is rather undermining of even present-day common sense, which has moved on somewhat since the 13th century, and hence is undermining of Aristotelianism.

It’s highly questionable that "common sense" has moved on, unless you take "common sense" in the quasi-facetious sense of Einstein's quip about prejudices laid down before the age of eighteen. I mean something a bit more serious than that; not something merely taken for granted, but sound practical judgement. Common sense tells you that you exist — or that other people exist, even if you haven’t worked through a philosophical demonstration (or even if you have); that 1+1=2, even though you haven’t studied Russell and Whitehead’s proof. That was true for Aristotle, and it is true today.

Of course, Aristotelianism isn’t grounded on common sense anyway. Some views entail that you are “nothing but” a pile of dead, unconscious atoms, say. If the self as an illusion or believing that you have no beliefs, etc. were the only possible logical explanation, we might be stuck with that… but not when there is a system that supplies a perfectly rational and consistent explanation of how you can in fact be what you seem to be (and the atoms to boot). The big deal about Aristotelianism and common sense is not its foundations, but its results; that it’s a system that can actually explain why common sense works, instead of sheepishly having to explain it away as something deluded.

I would have thought that the history of ideas teaches us that little can be set in stone. Arguably, the work of Bolyai, Lobachevski, and Gauss did undermine Pythagoras, at least until Euclidean geometry was reclaimed and reset in a newer, wider context.

As GRodrigues pointed out, it did no such thing — what could that even mean? That in the nineteenth century the square of the hypotenuse suddenly stopped equalling the sum of the other squares? That 3²+4² only approximately equals 5², and Pythagoras just didn’t have an accurate enough ruler to notice? Lobachevski added to our mathematical knowledge, but certainly did not remove anything that had already been established. It's not "Pythagoras's Conjecture" or "Pythagoras's Suggestion”, it is a proof, and not something that will ever be overturned.

Now obviously, not every claim reaches the level of proof. For Aristotle to say, “Chocolate ice-cream is better than strawberry", does not commit contemporary Aristotelians. Some claims are opinions or conjecture. Scientific claims are of their nature provisional — they are not mere opinions, but we know they will be improved and refined. You may see obsolete scientific examples quoted to show how much the Mediaevals didn’t know, but they knew that they were only examples, and subject to change.
Of course Aquinas didn’t know about genetic mutation, but he didn’t have to, because wasn’t staking any proofs on the details of such a claim. In fact, he famously throws in a line about “new species, if any such there be…” because he was completely aware that despite the “consensus science” of his day, biology is not certain in the way mathematics is. That’s why it’s dangerous to brag about what they “didn’t know” — Aquinas didn’t know biologists were going to change their minds and come up with evolution, but he was ready for them anyway. It turns out that the actual foundations, that parts that are supposed to be solid, in fact are, and the claims which can be defeated are not that important. There’s no merit in dismissing the ancients and mediaevals because “we know things” if one doesn’t make the effort to take them seriously and study what they actually said.

David Brightly said...

Thanks, gentlemen, for the responses. GR says,
The only way your claim would have any weight would be if we already knew that the mathematical descriptions as provided by the scientific theories are exhaustive of reality. But merely assuming that is question begging and is part of the dispute itself.
I might reply that the Aristotelian equally begs the question in assuming that the apparatus of cause, act, potential, etc, applies throughout reality. But rather than unproductively accusing one another of begging the question let's accept that both viewpoints are in effect axiomatic systems of thought. The question is then which, if either, does the better job of capturing reality? For many, the scientific image fails utterly to engage with what are the most important aspects of reality for us humans, and cannot even in principle address them. But this doesn't mean that Aristotelianism, as a theory of the manifest image, wins the race. As it turns into the final straight of the twentieth century it collides with discoveries about reality that no one could have predicted. These discoveries reveal a part of reality so far removed from common sense that axioms rooted in common sense and intended to characterise it just cannot reach it. Or so say I. The charge is not that bits of ancient and medieval science are wrong. Rather that the metaphysics is no longer up to the job.

On Pythagoras, I think GR gets things just right:
But the fact is that it [the work of Bolyai, etc] did not undermine Euclidean geometry; what it did undermine was the belief that there was, or could be (*), no other geometry other than the Euclidean...
This makes my point. For this belief was the common sense view from ancient times to Kant and beyond.

Anonymous said...

David Brightly: For this belief was the common sense view from ancient times to Kant and beyond.

I think you’re getting too hung up on this “common sense” thing. You can’t simply apply Sellar’s “manifest” label to Aristotelians as though they would accept his dichotomous characterisation of the so-called “scientific image” as a predominating rival. To get anywhere with that, you would first have to show that it does not, in fact, capture reality, and furthermore that it cannot be extended in any way to cover new knowledge. GRodrigues clearly noted that the discovery of non-Euclidean geometry did not overturn any proven mathematics; it merely showed that it was wrong to take it for granted otherwise. And of course everyone didn’t take it for granted — mathematicians were investigating the issue for centuries, which is how they managed to discover these other geometries in the first place. “Aristotelianism” did not stop short with the death of Aristotle, and supposing that any new discovery could warrant throwing out proven fundamentals is as silly as throwing out the scientific method because Newton didn’t know everything.

These discoveries reveal a part of reality so far removed from common sense that axioms rooted in common sense and intended to characterise it just cannot reach it. […] Rather that the metaphysics is no longer up to the job.

But I already pointed out that that’s backwards: Aristotelianism isn’t rooted in common sense, as though Aristotle just looked out the window and wrote down the first thing that came to his head. As I said, it’s the other way around. The fact is, the rejection of various Aristotelian principles is what led to many of the problems that modern philosophy faces. Traditional metaphysics handles these “unpredictable” discoveries better than its competitors. Some of these discoveries may be surprising, but they nevertheless fit within the traditional framework, i.e. they lend it further support. That’s why modern philosophers are starting to rediscover Aristotelian concepts, sometimes explicitly, sometimes re-inventing them (“emergence” seems to be stumbling towards formal and final causes under a new name).

Let’s get some concrete examples on the table: name some specific fundamental premises of traditional metaphysics and some specific discoveries, and explain why you think they are somehow incompatible.

David Brightly said...

Mr G,
Does the Aristotelian machinery of form, substance, cause, act, potential, etc, offer us any insight or understanding of how matter behaves on the smallest scales and of how space and time behave on the largest scales?

It's in these domains that the problem is starkest. I think the answer is No, primarily because Aristotle can offer us only common sense and the world departs from common sense at these scales. The microscopic world is not the macroscopic world writ small.

But worse, Aristotle offers us no real insights into the everyday world, either. He merely echoes back to us in very general terms what we already know in more specific terms.

I'm sorry to bang on about common sense, but that is just what we are getting. Aristotle did a great deal of careful looking out the window and organising what he saw into general categories and principles. But lacking a telescope and a microscope, just to mention two of the huge range of scientific instruments of the seventeenth century and onwards, for the next two millennia, his unaided eye is no more insightful then any other man's.

Some specific points from your last:
* How then do Aristotelians respond to Sellars's dichotomy? This is really my original question.
* Of course it was taken as granted that Euclidean geometry was the geometry of space. There were no other candidates. What mathematicians were doing was trying to prove the parallel postulate from the other four.
* It is often said but I deny that the rejection of various Aristotelian principles is what led to many of the problems that modern philosophy faces. This is back to front. The problems are like weeds under a heavy mulch. The seeds are there all the time needing only a little thinning of the mulch to spring up.
* In what sense are Aristotelian fundamentals proven? They strike me more as axioms.

Anonymous said...

David Brightly: Does the Aristotelian machinery of form, substance, cause, act, potential, etc, offer us any insight or understanding of how matter behaves

If you’re asking what metaphysics tells us about physics, then the answer is that it tells us that questions about how matter behaves are the job of physics. (Of course metaphysics as a foundation for the more specific sciences does give us the conceptual tools to go about answering those questions.)

because Aristotle can offer us only common sense and the world departs from common sense at these scales. The microscopic world is not the macroscopic world writ small.

On the contrary, common sense would tell you that the microscopic is not the same as the macroscopic! Even though I gave a definition of common sense and explicitly noted that in an Aristotelian context it cannot be taken to mean instinctive or culturally-influenced assumptions, you are still using it in the wrong sense. So let’s avoid the phrase “common sense” altogether, as it’s clearly too confusing. I’ll put it this way: is it remotely plausible that a system that’s been developed over thousands of years by some of the greatest minds known to history really has nothing more to offer than what any average person could come up with off the top of his head? If you honestly think that Aristotle, Aquinas, Scotus, et al. were a bunch of dopes, well, you’ve obviously made up your mind and there’s no point trying to discuss it. Otherwise, we at least have to agree that whether Aristotelianism is right or wrong, it’s not trivial.

But lacking a telescope and a microscope, just to mention two of the huge range of scientific instruments of the seventeenth century and onwards, for the next two millennia, his unaided eye is no more insightful then any other man's.

So what? They let us see more things, and this contradicts which metaphysical principle...?

Anonymous said...

(continued…)
* How then do Aristotelians respond to Sellars's dichotomy? This is really my original question.

They deny it. It’s a false dichotomy — obviously, Aristotelianism includes and encompasses science: not only is Aristotle sometimes called the first scientist, modern science got its start in the wake of the Aristotelian resurgence in the Middle Ages. Subatomic particles are still described in terms of final and formal causes, they are composites of actuality and potentiality, etc. All that the evidence supports is that specific details of Aristotle’s physics need to be updated in light of new physical discoveries, but since Aristotelians have always acknowledged that scientific theories are subject to revision, this is hardly an objection. Physics had changed from Aristotle’s day to Aquinas’s, and it has continued to change. Ultimately, present-day Aristotelians respond by asking critics to look at what they actually believe instead of swallowing rumours and caricatures.

I deny that the rejection of various Aristotelian principles is what led to many of the problems that modern philosophy faces. This is back to front. The problems are like weeds under a heavy mulch.

On what grounds do you deny it? To assume that all systems face the same problems is to say that all systems are interchangeable, in which case telescopes would be no more or less of a problem for Aristotelianism than for its modern rivals. Take the “problem” of qualia: the “scientific” view is faced with the question of how to explain qualities given an exclusively quantitative view of the world. But for Aristotle qualities and quantities are equally real facets of a physical substance. That problem can’t exist for him. (You might or might not attempt to show there are other problems, but it should be obvious that at the very least they aren’t going to be the same problems.)

* In what sense are Aristotelian fundamentals proven? They strike me more as axioms.

The axioms are pretty rock-bottom, like “reality is intelligible”. (If you want to go around warning people you have accept reason to be an Aristotelian, then I won’t stop you.) There are conceptual schemas (like describing change in terms of act and potency). There are observed facts (e.g. self-evidently true claims such as “something changes”). And of course, there are arguments that work from these starting points without bringing in other, questionable, theories or observations — and therefore the conclusions of which are certain, like various mathematical claims are certain and not able to be overturned by any new observations (however “surprising” and unexpected).

Again, I ask: what specific discovery is a supposed deal-breaker for what specific Aristotelian principle?

David Brightly said...

Mr G,

it [metaphysics] tells us that questions about how matter behaves are the job of physics.

Subatomic particles are still described in terms of final and formal causes, they are composites of actuality and potentiality, etc

Admittedly quoted out of distinct contexts, but isn't there a tension between these statements? I grant there is a formalism for calculating probabilities, but I cannot see how final causation plays any role at all.

On the contrary, common sense would tell you that the microscopic is not the same as the macroscopic!

Really? Without knowing what the microscopic looks like? How can this be a 'sound practical judgement'?

Obviously I no more think Aristotle, etc, were a bunch of dopes than I think that 3²+4² only approximately equals 5², or that accepting reason is any onus. Let's be serious. What I will say is that when any thinker speaks as a metaphysician I have grave doubts that he is telling us anything of any depth. Aristotle was clearly capable of marshalling a great many of his own and other's observations into a coherent system of understanding of the natural world, and thoroughly deserves the accolade of 'first scientist'. But when he says something is a composite of actuality and potentiality he is just telling us in rather grandiose language that it is capable of inducing change in other things and undergoing change itself. Or so it seems to me. I might be persuaded otherwise, but I incline towards thinking Aristotelianism is right but trivial, within the context of the manifest image (an important caveat). I accept the point about secondary qualities. But I suspect that had he known more about the psychology of colour vision, scientist as he was, he might have thought differently.

On what grounds do I deny it? I think there are such deep problems in elucidating the common-sense (no apologies) notion of (efficient) cause that I'm inclined to think it cannot be foundational. I'm encouraged in this thought by its non-appearance in physics. This is not to reject an Aristotelian principle but rather to have investigated it more closely. This was open to thinkers from Aristotle's time onwards, so I've no idea why it wasn't taken up until the modern era. Hence I see efficient causation as an element of the manifest image but not the scientific.

Anonymous said...

David Brightly: Admittedly quoted out of distinct contexts, but isn't there a tension between these statements?

Sorry, I was unclear: physics investigates the specifics about the particular forms and behaviour of this or that particular kind of matter. Metaphysics provides the general framework for what forms or final causes are in general. It’s parallel to mathematics: it’s the work of physics to figure out which particular formula applies where, but math provides the underlying tools and explanations for how equations work apart from any particular physical application. That’s why Newton had to discover calculus in order to invent Newtonian mechanics (and Einstein after him had to (get Minkowski to) discover tensor calculus, etc.).

I grant there is a formalism for calculating probabilities, but I cannot see how final causation plays any role at all.

I don’t see how it can not. What do you think final causality is, exactly? To an Aristotelian, what you have just said is that you don’t see how subatomic particles have any behaviour, but of course that makes no sense.

>On the contrary, common sense would tell you that the microscopic is not the same as the macroscopic!
Really?


Yes, the microscopic really is different form the macroscopic: one’s micro, and one’s macro. To assume to that the microscopic world behaves exactly the same as the macroscopic is unwarranted. They might be similar, or they might not, but one at least has to look into the matter first.

What I will say is that when any thinker speaks as a metaphysician I have grave doubts that he is telling us anything of any depth. […] But when he says something is a composite of actuality and potentiality he is just telling us in rather grandiose language that it is capable of inducing change in other things and undergoing change itself.

Starting from act and potency gets you demonstrations of God’s existence, which ought to be profound enough to start with. And it’s not “grandiose”, it’s just technical vocabulary. Every discipline has it, and often the terms have “trivial” everyday counterparts. Obviously a casual and superficial view of Aristotelianism is not going to reveal anything very deep; that brings us back to the Aristotelians’ response that people need to study their views seriously if they want their criticisms to be taken seriously.

(Conversely, the “scientific method” says to get an idea, try stuff, and see what works… isn’t that trivial enough to dispense with in these advanced modern times? Especially given the number of times that “science” has been wrong!)


[continued...]

Anonymous said...

But I suspect that had he known more about the psychology of colour vision, scientist as he was, he might have thought differently.

Contemporary Aristotelians know more about visual psychology, and they don’t. If you don’t understand what their position is, and don’t have a specific objection, how can you judge one way or the other?

I'm encouraged in this thought by its non-appearance in physics. […] so I've no idea why it wasn't taken up until the modern era.

I don’t know what exactly you mean wasn’t taken up, but as GR explained above, "causality" certainly does appear in physics. Of course it may be implicit, just as the absence of the term “universe” in biology does not mean that a universe is superfluous to biological organisms! It is precisely because it is so necessary that it can be taken for granted. But an even better reason why the word “cause” might not turn up often in physics is because (again) physics is about particular causes, and so it uses a vast array of terminology that refers to specific subtypes of causes. For example, the nearest physics textbook to hand is replete with terms like “pushes”, “pulls”, “exerts”, “displaces”, “applies [torque]”, “produces”, “conducts”, “scatters”, “refracts”, “repels”, “attracts”, “transforms”, “stimulates”, “emits”, and many other highfaluting synonyms for “causes”. Of course, it does occasionally mention “causality” by name, such as when it says, “the laws formulated by Newton are referred to as causal laws”. (It goes on to note that this “deterministic view of the universe had to be rejected by scientists in the twentieth century” — “deterministic”, not “causal”.)

Of course, it's full of other metaphysically-begotten terms, such as "matter", "substance", "potential", "form[ula]s", and so on. (A section under quantum mechanics is even entitled "Philosophic Implications".) Aristotle's fingerprints are all over science, even modern science, whether explicitly or implicitly, whether modern scientists are aware of it (and a few of them are!) or not.

David Brightly said...

Mr G, Thanks for bearing with me on this, and apologies for the delayed reply. It's a great help in getting my thoughts clear. Let's go back to what GR said at May 20, 2015 9:43 AM:

The "scientific image" does not get by without it [cause]. At all. What does happen is that in the *mathematical formulation* of the physical theories, words like "cause", "substance" and "essence" do no do any work. Nor could they, because these are precisely the aspects of reality that cannot be captured by the mathematical formalisms. But that does not mean that these, or their functional equivalents, do not play any role. Of course they do, because scientific theories aim at give a rational understanding of the world, not merely a mathematical description of it useful for prediction purposes.

This looks like an argument, using 'because' twice, but it's a nice, bold statement of the Aristotelian viewpoint. If I can summarise:

1. 'Cause', 'substance', 'essence', etc, denote aspects of reality.
2. These aspects are not and cannot be captured by mathematical formalisms.
3. A rational understanding of the world requires more than mathematical descriptions.

I say that we are not compelled to accept any of these claims. Against (3) we can say that the atomic world is known only through mathematical descriptions. Of course these descriptions require interpretation, but then so does any linguistic expression. Against (1) and (2) in combination we can say that vestiges of 'substance' and 'essence' can be found in the atomic world. There do seem to be individuals of kinds characterised by fixed properties, but all this is captured in the mathematical description. It's also worth noting that the notion of 'individual' that applies is not exactly the everyday one: in systems of identical particles it's not possible in general to re-identify individuals over time. The notion of 'the same one' evaporates. This in turn makes it hard to see 'cause' as a relation between individuals, at this level.

But I may have got 'substance' and 'essence' wrong. It seems to me that these words behave like the undefined terms of an axiomatic system, just as 'point' and 'line' function in Euclid. A parallel (sorry!) between metaphysics and mathematics perhaps?

You are right to point out the many terms in macroscopic physics that are freighted with 'cause'. What we find macroscopically is the aggregated result of myriads of acausal microscopic effects. We carve the world up into macroscopic individuals (leaving behind an undifferentiated 'bulk' that is not paid much attention). The existence of these aggregations has no impact on the form of the descriptions of the local microscopic effects. When we try to give an account of the world in terms of individual aggregations, and nothing else, we inevitably arrive at the notion of 'cause' as a relation between the individuals. In this we are inspired by our introspected sense that we can influence the behaviour of our fellows by speech and act and contrivance. It's quite understandable that we resort to this language to convey to others what happens in macroscopic physical situations or experiments, and to help them to understand it.

Anonymous said...

I don’t mind about the delays if you don’t!… so if you’re still reading:

David Brightly: Against (3) we can say that the atomic world is known only through mathematical descriptions.

If that were the case, then (particle) physicists would be mathematicians, not physicists. But even theoretical physicists claim that the symbols in an equation are stand-ins for physical things (“substances”) such as electrons, neutrinos, etc. Plus their colleagues in experimental physics put quite a lot of work into detecting actual particles. If we never detected them, we wouldn’t have any knowledge about them at all.

Against (1) and (2) in combination we can say that vestiges of 'substance' and 'essence' can be found in the atomic world. There do seem to be individuals of kinds characterised by fixed properties

That sounds like an Aristotelian substance to me, not a vestige. The fact that one electron may be intrinsically indistinguishable from another is exactly the sort of issue that “substances” and “accidents” capture, so it turns out they are just as essential to describing the subatomic realm as any other. The fact that it may be difficult (even impossible) in practical terms to identify which electron is which is not a problem on the metaphysical side, any more than a difficulty in identifying an individual person in a crowd all dressed the same. And even if you insist that it’s not merely a question of epistemology, that an electron isn’t “the same one” at all — all that means is that subatomic particles go into and out of existence very easily… which we already knew, and which is merely a new example of good old Aristotelian act and potency.

It seems to me that these words behave like the undefined terms of an axiomatic system, just as 'point' and 'line' function in Euclid. A parallel (sorry!) between metaphysics and mathematics perhaps?

I’d agree that in some sense they are axiomatic, though of course an Aristotelian (or Euclidean) textbook will certainly provide technical definitions of a substance or essence (or line or point). And the definitions are very broad, so that they are able to encompass discoveries no matter how unexpected. The fact that physicists can and do have a lot to say about what electrons are like means they assuredly qualify as “substances” of some sort.

It's quite understandable that we resort to this language to convey to others what happens in macroscopic physical situations or experiments, and to help them to understand it.

I take it you mean that our talk of cause and effect is merely an attempt to describe the alien world of elementary particles in language we can understand. But it wouldn’t help to understand anything if it didn’t bear some resemblance to how particles actually behave, which means some form of cause and effect does apply (even if not exactly as we think of it “macroscopically”). Which is again why it’s important to study what metaphysicians mean when they use those terms in the technical sense. (A physicist uses the word “light” in a way that differs from the everyday use of the word, but that doesn’t mean that it’s not a legitimate use, or that it bears no relation to light in the ordinary sense.)

Of course, even the everyday sense of cause and effect is clearly applicable, at least roughly: what is an experiment other than setting up some “cause” in order to, er, cause a particular “effect”? If physicists could not set up particles in some structured way and then observe what happens, then quantum physics wouldn’t be a science. I don’t know what it would be, but clearly it is a science and it’s precisely because we can do experiments, because we can cause certain quantum events, and measure their effects, that we can even talk about these particles in the first place. If cause and effect really didn’t apply, then there would be no meaningful way to say what it is that doesn’t apply to the things we can’t be talking about!

David Brightly said...

Well, theoretical physicists are mathematicians. A particle physics experiment produces a vast pile of numbers which can only be made sense of via mathematics. It's not as if we know these things by acquaintance.

As I see it, the Aristotelian faces the problem of explaining how his concepts of 'substance' and 'cause' as a relation between substances can accommodate,
(a) indeterminacy of location and momentum;
(b) indeterminacy of identity of two distinct kinds---bosons and fermions;
(c) quantum entanglement.
I'm not aware of a latter day Georges Lemaître who has done this. After all, if we really are dealing with 'good old act and potency' and the definitions are 'broad enough to encompass discoveries no matter how unexpected', then it can't be a difficult job, surely? But it is difficult. How, for example, is the notion of 'independent existence' built into the technical sense of 'substance' going to handle the phenomenon of entanglement? Not that other philosophies of physics have answers. It's just that the claims for Aristotelianism appear singularly hubristic. I don't see how the indeterminacy of identity is to be explained by the ease with which 'subatomic particles go into and out of existence'. Nor is it a question of technical definitions. By 'light' physicists mean just the same as the rest of us. They just know a lot more about it.

I'm really making two claims here. First, the above claim that the Aristotelian framework cannot accommodate the quantum world. It simply is not the case that 'we can cause certain quantum events'. All it seems we can do is set physical systems into a state (this is usually done by a process of filtering out the undesired states) in which the quantum processes may or may not take place. More controversially perhaps, I'm saying that we do not have to accept cause and effect in the macroscopic, everyday world, either. Clearly our ordinary thinking about the world involves what we call 'cause and effect'. But if the metaphysician is saying that in some sense we must think in these terms, that it is the only possible way because that is the way things are, then he is in danger of over-reaching himself. For we have found alternatives that work very well indeed.

Anonymous said...

David Brightly: Well, theoretical physicists are mathematicians.

Sure, anyone who can add is a "mathematician". But you know what I meant.

if we really are dealing with 'good old act and potency' and the definitions are 'broad enough to encompass discoveries no matter how unexpected', then it can't be a difficult job, surely?

I said the principles were "good", not "trivial". (Newton's laws of motion are pretty simple, so it can't be a difficult job for you to solve the three-body problem... right?) Of course, if we understood fully what was going on, explaining it in Aristotelian terms might not be that hard; the catch is that we don't fully understand everything that's going on. Get back to me when physics has figured out all the details.

How, for example, is the notion of 'independent existence' built into the technical sense of 'substance' going to handle the phenomenon of entanglement?

What precisely about quantum entanglement is it that you think necessarily makes an Aristotelian interpretation flat-out impossible? Two substances have various properties that are related in certain strict ways. Or perhaps you want to say that the entangled substances are really “one” substance in some sense, or something like that — that’s fine, there’s no problem in starting with one substance, and then, say, ending up with two separate substances after wave-function collapse. But then again, you didn’t say quantum entanglement; maybe you were thinking of yarn-ball entanglement. If Aristotle claims that two balls of yarn are separate substances, is he disproved because it’s “difficult” to disentangle them?
Likewise with questions of identity: there is something there (at the end of the day, “substance” is just the technical term for “thing”) — what’s the alternative, that there is nothing there? If something comes into and then out of existence, then there is no identity, because the thing isn’t there any more… the situation may be complex and confusing, but it’s still a situation about something. Clouds are hard to identify too: where exactly does a cloud stop? Appealing to QM sounds all spooky and confusing, but the metaphysical questions apply just as much to things we can see looking out the window, like clouds and balls of wool. (In fact, clouds were even harder for Aristotle, because he wasn’t an atomist, and yet nobody considered this a fatal Achilles’ heel.)

By 'light' physicists mean just the same as the rest of us.

In everyday use, “invisible light” is an oxymoron. In the jargon of physics, on the other hand, “light” means “electromagnetic radiation”, so it’s not a contradiction at all.

It simply is not the case that 'we can cause certain quantum events’.

Of course it is. Otherwise you’d be claiming that it is literally impossible to do quantum experiments, in which case QM wouldn’t be physics or science at all. Oh, maybe you mean that we cannot set up any arbitrary quantum state at will, which is quite true (and not what I claimed). There certainly are some sets of circumstances that can be set up and triggered and produce a certain class of expected results, because that’s exactly what an experiment is, and physicists obviously do perform these experiments, and what’s more they can repeat them, because that’s what science consists of.

Anonymous said...

But if the metaphysician is saying that in some sense we must think in these terms [of cause and effect], that it is the only possible way because that is the way things are, then he is in danger of over-reaching himself. For we have found alternatives that work very well indeed.

But we haven’t, as I indicated in my previous responses. I don’t see how anything you have said has demonstrated otherwise, other than in the sense that I could claim we don’t need to believe in the third dimension because we can take two-dimensional photographs of everything.

It seems that you are taking the “manifest” view of Aristotelianism as a definition, such that it requires everything to be considered in terms of billiard balls because that's all Aristotle could see out his window. But this is to deal with a caricature. The Aristotelian system that was developed up through the Middle Ages was quite sophisticated and drew a vast set of subtle distinctions that got left behind with the switch to modern “science”-based metaphysics. Newtonian physics was terrific, and it viewed everything as billiard balls, so we should do philosophy that way, right? Except that billiard balls don’t work so well when it comes to Relativity or QM. Some seem to think that Aristotelianism came first and thus must be even more primitive than billiard-balls, but it’s actually far more able to deal with these discoveries. Which brings us back to the original point that Aristotelians reply by saying people need to actually find out what they really believe.

David Brightly said...

Of course, if we understood fully what was going on, explaining it in Aristotelian terms might not be that hard; the catch is that we don't fully understand everything that's going on. Get back to me when physics has figured out all the details.

This captures the issues nicely. As I understand it the conventional wisdom is that quantum phenomena cannot be understood in classical terms. It's not that we are missing some details. There are no more details there to be missed! That's why the whole business is so perplexing to our ordinary ways of thought. The only way we have of understanding it is through abstruse mathematics.

...there’s no problem in starting with one substance, and then, say, ending up with two separate substances after wave-function collapse.

The problem here is that the notion of 'substance' becomes non-local and capable of transmitting signals faster than light.

...what’s the alternative, that there is nothing there?

The alternative is that there is something there but it's not a thing, if that's not too paradoxical! In other words, 'thingness' is the wrong concept to apply. But it seems it's the only concept Aristotelianism has.

But we haven’t [found alternatives that work very well indeed]

We disagree then on the meaning of 'cause'. Which is where we began. The alternatives to cause that I have in mind are the equations of mathematical physics. They give us a different way of understanding the phenomena. A different metaphysics, if you like.

It seems that you are taking the “manifest” view of Aristotelianism as a definition

I stand accused of 'vulgar Aristotelianism'. Who should I read to enlighten me? By the way, are clouds substances? I would have thought not since they lack any unifying principle. Like heaps of sand they have no integrity.

Anonymous said...

David Brightly: conventional wisdom is that quantum phenomena cannot be understood in classical terms. It's not that we are missing some details.

These are two different things. I didn't mean to suggest that we need to wait for physicists to figure out whether this indeterminate object "really is" just a fermion or a boson so we know what substance we're dealing with. I just meant that physics isn't a closed book, so future theories may well suggest different or better interpretations. But Aristotle doesn't have any problem if the end result is still something "indetermintate" with respect to being a boson or a fermion (or a wave or a particle, or any other quantum puzzle) — there is no metaphysical reason per se to rule out a substance that simply has curious "superimposed" properties of both. There is a reason to rule that out in classical (i.e. Newtonian) physics. But that has nothing to do with "classical" (i.e. Aristotelian/Scholastic) metaphysics.
Perhaps that's part of the confusion: thinking that "classical" just means anything before twentieth-century physics; but modern physics seems puzzling precisely because science rejected various traditional classifications and distinctions. Modern discoveries are just pulling them back in. So when you appeal to the necessity of abstruse mathematics as an "alternative" to causality, Aristotle just shrugs and says, of course, that's what formal causality has been all along. When modern science seem to "disprove" causality, what they are ruling out is "classical" causality, which was (re)defined to be the narrowest possible sense [that classical science demanded]; on the other hand, Aristotelian causes can cope with it because they were defined in the widest possible sense.

the notion of 'substance' becomes non-local and capable of transmitting signals faster than light.

No problem. Aristotelian metaphysics (unlike classical physics) does not require that substances be non-local. It would certainly be interesting if that turned out to be the best explanation, but it wouldn't destroy the distinction between substance and accident, or between form and matter, etc., etc.

The alternative is that there is something there but it's not a thing, if that's not too paradoxical!

I think it surely is too paradoxical! But again, the paradox is resolved by remembering that the Aristotelian concept of a "thing" is broader than the "classical" one.

By the way, are clouds substances? I would have thought not since they lack any unifying principle.

Yes, I was going to say something like that... that is, it clearly has a sort of unity (or we wouldn't be able to point out a "cloud"), but I expect most Aristotelians would consider them to be artifacts. But in principle, there could be cloudy substances, and if elementary particles are "fuzzy" in that kind of way, it doesn't preclude their being fully Aristotelian substances. (But there's also the possibility that they are indeed not substances, which simply relocates the substances involved, even if it's not obvious to us "where" they are. (There must be a better way to put that. But consider even a Bohmian system where the particle is one substance, and there is another substance, the pilot wave. Or maybe they are both parts of a single substance. Or maybe the pilot wave is actually an artifact...)

I stand accused of 'vulgar Aristotelianism'. Who should I read to enlighten me?

Heh, I like Ed Feser (e.g. Aquinas) since he explicitly explains things in a contemporary context. Copleston's History of Philosophy is also good, though necessarily compact. (Adler's Ten Philosophical Mistakes might be helpful for context... it can take a sustained attempt to "unlearn" thinking in modern terms before the traditional view suddenly clicks.)

David Brightly said...

there is no metaphysical reason per se to rule out a substance that simply has curious "superimposed" properties of both.

The law of non-contradiction perhaps?

So when you appeal to the necessity of abstruse mathematics as an "alternative" to causality, Aristotle just shrugs and says, of course, that's what formal causality has been all along.

'Form' has no precise meaning. In the context of 'formal cause' it is suggestive of many ideas including 'shape', 'plan', and 'orderedness' as opposed to 'chaos'. But mathematics can offer a characterisation of chaos, so I would be wary of identifying mathematics with 'formal causation'.

No problem. Aristotelian metaphysics (unlike classical physics) does not require that substances be non-local.

Do you mean 'local' here? Given A's animadversions on the actually infinite I doubt he would contemplate a non-local, ie infinite, substance.

I have read Feser's The medieval principle of motion and the modern principle of inertia. He says,

Second, that some fundamental material substances (basic particles, say) exist and behave in accordance with such laws can also never be the ultimate explanation of anything, because we need to know, not only how such substances came into existence, but what keeps them in existence. For as compounds of act and potency, they cannot possibly account for themselves, but require something outside them to actualize them at every moment,

and then refers to his Existential Inertia and the Five Ways which is behind a paywall. As I understand these terms something that is not changing does not require an actualiser of its potency. Is that right?

Anonymous said...

David Brightly: The law of non-contradiction perhaps?

What contradiction? There's nothing contradictory about a substance having multiple overlapping properties, or multiple parts with different properties, or properties that behave in different ways under differing conditions. The only way to get a contradiction is to impose some other condition, such as assuming that the objects in question are restricted to, say, acting like Newtonian billiard balls. Historically, of course, physicists assumed that would be the case, so they were caught off guard by QM and it got labelled as "weird" and "spooky", but it isn't really. It's just complex. In fact, one of the motivations for (supposedly) replacing Aristotle was that the system was overall too complicated, but of course it had to be since the metaphysics was meant to explain any possible world with any possible laws of physics, and on top of that it turned out that actual physics needed the more complex system anyway!

But mathematics can offer a characterisation of chaos, so I would be wary of identifying mathematics with 'formal causation'.

A form is that by which something is the particular thing or kind that it is. That entails some sort of opposition to "chaos" (although probably in a trivial way, since "chaos" is simply the absence of order or form). I suppose that when you say mathematics can characterise chaos, you are thinking of Chaos Theory, which obviously does not use the term in its strict Platonic sense, since that would make it the study of order that is unordered. Rather, the whole point of chaos theory is that certain things that seem highly disordered can in fact be the result of highly ordered patterns — even, ultimately, quite simple patterns (that is, it is possible for a elegant and simple equation to produce a highly irregular [at least to our senses] pattern). In practical terms, chaotic systems can lead to unpredictable results because computational accuracy becomes infeasible (e.g. the infamous butterfly-flapping-its-wings)... or conversely, it gives us the tools to approximate very complex systems with much simpler models. But it works mathematically because the systems are deterministic.

Anyway, formal and final causes include (but of course are not limited to) quantitative properties, i.e. anything that can be described in mathematical terms, as modern physics can. So whatever "strangeness" lies in QM, insofar as it can be quantified — and it all can, because that's how the science is done — is describable by formal causes.


Given A's animadversions on the actually infinite I doubt he would contemplate a non-local, ie infinite, substance.

Yes, I meant "local". But "non-local" does not mean "infinite". Besides, infinity is only problematic in certain ways (basically, "completed" infinities, which means an infinity we got to the end of, which has a rather obvious problem). Not that contemporary Aristotelians are limited to calculus, etc. as it was (un)known in Aristotle's day, but Aristotle himself believed that the world had no beginning, so clearly some sorts of real infinities are not a problem.


As I understand these terms something that is not changing does not require an actualiser of its potency. Is that right?

Right, there is nothing actualising those particular potencies. I would say the argument goes roughly like this: something that has potencies is not a necessary being (or it would be pure act); but a contingent being has the potential to not-exist at any moment; therefore there must be something (not itself) that is causing it to be an actual being at each moment of its existence.

David Brightly said...

It's worse than mere complexity. No 'thing' can be both a wave and a particle. Particles are point-like, waves are spread out. Contrary properties. Hence the weirdness. The problem is to find some way to understand this.

Not Chaos Theory. Any system with randomly occurring states of equal probability is orderless, eg, coin tossing or die throwing, but very simply characterised mathematically. Such systems are formless yet have mathematical form.

If things have the potential to fall apart then surely it's the absence of an actualiser of such potentials that keeps them in existence, not a presence?

Anonymous said...

David Brightly: Particles are point-like, waves are spread out. Contrary properties. Hence the weirdness. The problem is to find some way to understand this.

No, that's just complexity. The relationships, the mathematics, aren’t as simple as if we had just a billiard ball or just a surging ocean, but it’s only “weird” relative to the expectation of such. Of couse elementary particles arent “both” waves and particles literally. They are like waves in certain ways and like particles in certain other ways. And we do have a way to understand this: they follow precise mathematical rules that have been discovered. What we can’t do is imagine it, because imagination means visualising colour and shape and size in ways that simply do not apply. So particle physics is harder to do than other things, that’s all.

That’s why I keep coming back to specifics. “Weirdness” is just a statement about our psychological reactions. To show an actual problem, you would need to show me some actual equations, describe an actual experimental setup, that results in some contradiction. And, of course, explain exactly what it is a problem for. (This is what Bell did with his famous inequality concerning locality.) That’s what is required, not vague hand-waving about particles/waves/won’t they make up their minds!

(A simple double-slit experiment will suffice to demonstrate a problem if the criterion is matching the classical idea of a particle or wave. That successfully disposes of the notion that all physics is classical. Now, I’m confident that you cannot pose such a problem for Aristotelian metaphysics because metaphysics is the foundation for what it is to have laws of physics at all; it is the most generally necessary framework for thinking about formulas and experiments in the first place, so by giving me formulas and experiments you are necessarily presupposing just what Aristotelian metaphysics supplies. The best you could hope for is some sort of reductio ad absurdum that disproves the possibility of physics altogether. But since I’m quite confident that physics isn’t impossible, I’m confident that nobody will be able to provide such a demonstration.)

eg, coin tossing or die throwing, but very simply characterised mathematically. Such systems are formless yet have mathematical form.

Then they aren't formless. Obviously they don't have every form, they are described by some patterns and not others. To say that a coin-toss is random just means random with respect to some other variable… insofar as “orderlessness” is a lack of form, sure, there is no form, say, of “likelihood of heads related to the phase of the moon”. The mathematics describes the forms they have and doesn’t the forms they don’t. But that is just what formal causation is all about.

If things have the potential to fall apart then surely it's the absence of an actualiser of such potentials that keeps them in existence, not a presence?

That absence is indeed what explains not having the different, fallen-apart parts separating and floating around individually. But rather here the point is that likewise, the thing has a potential to stay together, and that must be actualised just as much as any other potential. I haven’t read that paper either, but Feser has lots of articles on his site about act and potency, and typically he describes this in terms of essence and existence. (If something necessarily existed — if it’s essence entailed existence — then it would be pure act, so something that has act and potency must have its essence separate from its existence. This is just noting that we can think of, say, the idea or form or essence of a unicorn without any unicorns actually existing, i.e. they’re contingent beings. But then if a contingent being does exist, it must get its existence from something else.)

David Brightly said...

Aside: Why does Aristotelianism seem to engender such confidence in its adherents? I am suggesting that experimental results have got us scratching our heads over what we understand by 'matter' and 'substance'---and this isn't just a personal quirk of mine---but this is breezily dismissed as mere 'complexity', and anyway, 'you cannot pose such a problem for Aristotelian metaphysics because metaphysics is the foundation for what it is to have laws of physics at all'. Ed Feser is fond of this formula too. Galileo has my sympathies! I have to say that I'm not at all clear as to what kind of statements or principles would form such a metaphysical foundation. But what I'm suggesting here is that Aristotle's ideas can't be adequate because, amongst other reasons, the fundamental notion of 'substance' runs out of meaning in the very small. But this is to cycle back to our starting point. Anyway, here is a proposal for a foundational metaphysical principle: Physical laws have to appear the same in all inertial frames. This is because of the metaphysical principle that 'absolute rest' is meaningless. Coupled with the experimental fact of the constancy of the speed of light it says that the equations of physics must be Lorentz invariant. This is not in Aristotle for obvious reasons. Quite the reverse, in fact.

You say 'they [particles] follow precise mathematical rules that have been discovered'. Well, I'm not sure that 'they' do, because I'm not sure that 'they' makes any sense. At high energies even the number of 'them' from moment to moment is subject to uncertainty. This is my fundamental point. What QM gives us is a set of rules for calculating the probabilities of experimental outcomes. It's not clear that the mathematics is about the properties of individual substances as it is with classical mechanics. It's couched in terms of operators acting on some more basic evolving entity. See below on Bell's theorem.

I don't know if anyone has attempted a double slit experiment with electrons. But we know that electrons display diffraction effects as well as particle effects. The problem is that the particle effects point towards a size (electrons are little bits of matter, right?) that is smaller than the smallest degree of spread-out-ness required by the diffraction effects. This is seen as contradictory, and not just by me. You admit that 'What we can’t do is imagine it, because imagination means visualising colour and shape and size in ways that simply do not apply.' Well, why don't they apply? The simple and reasonable answer is that whatever 'they' are, they cannot be conceived as substances, and this fits well with what we know of their other odd behaviour. But Aristotle may be forgiven for not foreseeing this.

Bell's theorem says that for certain measurements QM predicts different results from 'local realist' theories. A realist theory says that entities are characterised by sets of properties (possibly 'hidden', but determining the motion) with definite, if changing, values at all times. The latter sound just like Aristotelian substances. But experiments back up the QM predictions. So again we pressed to revise our conception of substance.

I accept that the language of act and potency nicely accounts for change in things. But isn't there a logical difficulty in accounting for the continued existence of things in such terms? For if a thing's hanging together is explained by the actualisation of a potential to hang together then it can no longer have that potential. The hanging together cannot be both potential and actual simultaneously. At least, that's how I understand how these terms are to be used. Furthermore, before the potential is actualised what is the 'it' that has the potential? Is it the mere parts, collectively?

Anonymous said...

David Brightly: Why does Aristotelianism seem to engender such confidence in its adherents?

Perhaps because the low awareness about it means they are usually defending basic, straightforward, solid aspects that merit much confidence. It surely beats the oddly determined confidence of sceptics, no?

I am suggesting that experimental results have got us scratching our heads over what we understand by 'matter' and ‘substance’

Yes, and I’m suggesting that your itchy head is not a legitimate reason for an Aristotelian to get worked up over anything. Of course lots of people may be in a similar boat — lots of people are wrong about lots of things. You haven’t actually produced an argument that shows any contradiction or inconsistency in the Aristotelian view, or shown how any physical facts (as opposed to interpretations) are flatly impossible to describe in Aristotelian terms. If you don’t have a precise argument from premise to conclusion showing how the Aristotelian view can’t work, then why do you think that it must be mistaken? In fact, why would you even suspect that it’s mistaken? (Do you have any grounds for thinking that people who retain a confused or sceptical position are experts in Aristotelian metaphysics such that they must have such proofs themselves?)

Galileo has my sympathies!

Well, Galileo certainly sometimes let his ego get the better of him, but presumably you are thinking of his exasperation with people who lazily recited Aristotle instead of working with new and better evidence as it became available. (From chaps like Kepler. (But don’t tell Galileo’s ego.)) Of course Aristotle would be the first to rebuke those people himself, for ignoring his own advice about following evidence! Fortunately, nobody is claiming that Aristotle was the last word in physics; only the first word in metaphysics. And Galileo no more refuted any of his metaphysics than Bohr did. In fact, the hand-waving references to “But Science says!” instead of looking through the metaphorical (or metaphysical?) telescope would put Galileo on the side of the Aristotelians in this case.

I have to say that I'm not at all clear as to what kind of statements or principles would form such a metaphysical foundation.

Which brings us full circle: you wanted to know what Aristotelians would say in face of alleged modern challenges, and they would say, “you’re not at all clear on what would form such a foundation.”
I wonder if we have here a confusion between “common sense” and “Newtonian physics”… the old Newtonian view is common-sensical (because it’s easy, or easier, to imagine), and Aristotle is based on common sense, therefore the Aristotelian view is just an even more primitive version of the Newtonian view, and modern physics has disproved the Newtonian view, therefore Aristotle is disproven a fortiori. Except the classical Newtonian view is not especially common-sensical (it’s common, because it’s so baked into our culture in many ways, but not necessarily into human nature); and the philosophical interpretation that metaphysics can simply and directly be read off Newtonian physics is in fact at odds with the traditional view. The overthrow of Newtonian assumptions is (in at least some respects) a vindication of Aristotelian metaphysics.

(cont'd...)

Anonymous said...

(...continued)
But what I'm suggesting here is that Aristotle's ideas can't be adequate because, amongst other reasons, the fundamental notion of 'substance' runs out of meaning in the very small.

If part of the definition of “substance” were “being very large”, then you would have a point. But it’s not, so I just don’t see where the supposed problem lies. Do very small things change? Physics says yes. Therefore, very small things are substances. The fact that a subatomic substance doesn’t act like something larger in no way indicates that it isn’t a substance; but all you have shown (or alluded to) is that elementary particles are not the same kind of substance as, say, billiard balls. Which is something nobody disputes (except perhaps for some late nineteenth-century physicists who weren’t Aristotelians anyway).

Physical laws have to appear the same in all inertial frames. […] This is not in Aristotle for obvious reasons.

That’s not a metaphysical principle at all, it’s a physical one. It is particular to the way physics happens to work in this world, but there could be different laws of physics, different systems in which “absolute rest” is entirely meaningful. And of course physical principles are obviously not in Aristotelian metaphysics.

At high energies even the number of 'them' from moment to moment is subject to uncertainty.

So what? The number of people in China from moment to moment is subject to uncertainly too. In fact, in practical terms, I am quite confident that nobody will ever get an exact count of the population of China, but that doesn’t mean that people are weird non-substances incapable of being characterised in Aristotelian terms, or that mathematicians are cocky to insist that arithmetic is certain and incontrovertible, indeed, that if arithmetic didn’t work, you wouldn’t be able to count things in the first place. The “some more basic evolving entity” you speak of just sounds like yet another substance. (If the problem is whether there is one substance or many, that’s fine — again, one substance is still a substance. Nothing you have said shows how there could be no substance present — i.e., nothing which is undergoing change. In fact the term “evolving” presupposes that, and similarly for the term “entity”.)


I don't know if anyone has attempted a double slit experiment with electrons.

Yes, back in 1927, three years after de Broglie predicted that particles like electrons should also behave like waves. Since it’s real, obviously it does not involve any actual contradiction; and since the realities (what can be observed in experiments or quantified in equations) fits into the old Aristotelian categories, it’s no counter-example. (Note that I refer the actual physics, not interpretations of the physics.)

Electrons (physical stuff, not hypothetical abstractions = material cause) with certain properties (mass, charge, spin, etc. = formal cause) can be generated by an electron gun (heated filament = efficient cause) and result in a interference pattern (mathematically distributed patterns = final cause). We have a substance (an electron which persists from gun to plate) with changing accidents (properties such as position and velocity) as its potentials are actualized (by the electron gun, interaction with the diffraction grating, reaction with the plates). Where’s the problem?

(cont'd...)

Anonymous said...

(...continued)
The problem is that the particle effects point towards a size (electrons are little bits of matter, right?)

No, you’re making too many assumptions. Once again, being a tiny little billiard ball having those effects might point at that, but since we all know that elementary particles are not little billiard balls, that conclusion simply isn’t warranted. Whether or not it is “natural” to imagine particles that way, we have to stick to the facts, not our imaginings. If that makes it hard for us to understand QM (and in many ways I grant that it does), it isn’t Aristotle’s fault.

'...imagination means visualising colour and shape and size in ways that simply do not apply.' Well, why don't they apply? The simple and reasonable answer is that whatever 'they' are, they cannot be conceived as substances

That isn’t reasonable at all (as I have indicated). The reasonable answer is that some things just are not the kind of thing that is [easily] imaginable. You cannot imagine what air looks like because air does not have any colour — not because air cannot be conceived of as a substance! An electron is not literally a tiny billiard ball, nor an ocean wave, so no, we cannot accurately imagine it to be one. Fortunately, human beings are rational animals, and can understand things with our intellects, not merely our imaginations, so we can understand what electrons are like even if we cannot picture them. (Of course, we do not understand everything about electrons. That doesn’t prove they aren’t substances either, only that they are complex substances capable of being partially understood.)

But Aristotle may be forgiven for not foreseeing this.

And Aristotle is the one who refrained from building any definitions into his metaphysics that presumed that any sort of “particle”, say, would necessarily have to be (imaginable) like a ball. His modern opponents could learn from his carefulness and restraint.



Bell's theorem says that for certain measurements QM predicts different results from 'local realist' theories. A realist theory says that entities are characterised by sets of properties

Notice how you dropped the word “local” there? The misleading implication is that Bell’s Inequality rules out a “realist” interpretation of QM… which would be rather odd, since Bell himself was a realist. He just wasn’t a local-realist. “Properties with changing values” actually sounds like either substances or accidents, but since both are Aristotelian, that’s good enough for me. But of course, every interpretation of physics has that, because (again) that’s just what physics does (otherwise, you couldn’t use math to do physics), regardless of whether it’s local or non-local.

(cont'd...)

Anonymous said...

(...continued)

Anyway, Bell provided a precise mathematical formulation of his theorem and explicitly described what an experiment would have to show (which was subsequently performed and demonstrated). I haven’t seen anything comparable concerning the claim that traditional metaphysics cannot account for anything in modern physics. I’ll give a specific example to the contrary: Lowe describes the case of a helium atom absorbing an electron, and then later emitting one, and claims that there is no fact of the matter about whether it is the same electron or not. First of all, he seem to take for granted that the superposition in the electron shell is not merely an epistemological limitation (where does he address a Bohmian approach?), but more importantly for our purposes, he seems to assume that the only possibilities are that the electrons are permanent and retain their discrete identity even inside the ion, or that identity is indeterminate. He never even alludes to what is probably the most obvious case for an Aristotelian: that the captured electron ceases to exist. Aristotle can, and I think is quite likely to, claim that there is no more original electron, only an electron-like part of the helium atom. But of course, people who don’t think in the classical tradition don’t think of virtual parts, so it’s not surprising that his argument will carry little to no weight with an Aristotelian. He’s not even addressing their position to begin with. (Lowe also explicitly notes that identify is not ruled out tout court, recognising that that would undermine the very argument he’s trying to make, but he doesn’t defend this rather ad hoc excuse.) And it seems to me all such arguments follow the same path: they start with assumptions that dismiss the traditional view and/or simply ignore it. But they never disprove it.



The hanging together cannot be both potential and actual simultaneously.

Right. Once the hanging-together at this moment has been actualised, that’s it. But at this moment, it is only potentially still together at the next moment; so something has to actualise that potential at the next moment, and the moment after that, i.e. something has to be actualising it at every moment that it exists.

Furthermore, before the potential is actualised what is the 'it' that has the potential? Is it the mere parts, collectively?

Also right. Strictly speaking, the thing in question cannot have any potentials if it doesn’t exist yet; its constituents may have the potential to bring something new into existence — the underlying matter would be the substrate for that change, wherein the matter was informed one way, and then takes on different forms… but matter itself is not a substance, only matter + form. And in terms of QM, that should suggest how problematic it is to try to “disprove” substances: any time a proposed impossibility comes up for a substance, it’s possible that the matter involved simply ceases being one substance and another substance(s) comes into existence. There’s no minimal level of continuity required, so any suggested grounds for throwing out substances in some particular case are going to be purely arbitrary.

«Oldest ‹Older   201 – 300 of 300   Newer› Newest»